Section 8 Infectious Diseases
577
SECTION 8 Infectious Diseases
578
CHAPTER 29
579
Endocarditis
580 Case: A 31-year-old man with pulsating nail beds A 31-year-old man with active intravenous methamphetamine use is
admitted to the hospital for evaluation of dyspnea. The patient reports a 1 week history of fever and drenching night sweats. Over the past few days, he has developed progressive shortness of breath. He
describes difficulty lying flat and has been waking up in the middle of Temperature is 39.2°C, heart rate is 106 beats per minute, and blood the night gasping for air. pressure is 110/38 mm Hg. The patient’s head is moving in a to-and- fro manner. Jugular venous pressure (JVP) is 18 cm H O. The carotid 2 and radial pulses are bounding. There is alternating flushing and blanching of the nail beds in concert with the cardiac cycle. There is a
3/6 early-peaking crescendo-decrescendo systolic murmur best heard diastolic murmur best heard over the third intercostal space of the left and hypothenar eminences (Figure 29-1A) and the pulps of the over the right upper sternal border as well as a 3/4 decrescendo sternal border. There are tender erythematous nodules on the thenar fingertips (Figure 29-1B).
FIGURE 29-1
What is the most likely underlying diagnosis in this patient?
What is endocarditis? What are the 2 general categories of endocarditis?
What is the relative prevalence of Noninfective endocarditis compared with that of Infective endocarditis (IE)?
Endocarditis refers to inflammation of the innermost layer of the heart (endocardium), usually involving the heart valves. Endocarditis can be Noninfective or Infective.
The overwhelming majority of cases of endocarditis are infectious in nature. The annual incidence of IE in the industrialized world is 3 to 10 cases per 100,000 persons. Noninfective endocarditis is comparatively rare, representing <5% of cases.1-3
581
Noninfective Endocarditis
What is Noninfective endocarditis?
Which heart valves are most frequently involved in nonbacterial thrombotic endocarditis? What are the clinical manifestations of nonbacterial thrombotic endocarditis? What are the treatment options for nonbacterial thrombotic endocarditis?
Noninfective endocarditis refers to the presence of sterile vegetations (primarily composed of platelets and fibrin) on the endocardium, most frequently involving the heart valves. It develops in association with a variety of conditions, including inflammatory and hypercoagulable states. Nonbacterial thrombotic endocarditis (NBTE), previously known as marantic endocarditis, is the umbrella term that refers to all forms of Noninfective endocarditis regardless of size or location of the vegetation, previous health of the involved valve, or underlying cause.4,5 NBTE may affect both normal and previously damaged heart valves. The mitral valve is most commonly involved (two-thirds of cases) followed by the aortic valve (one-third of cases). NBTE may involve both the mitral and aortic valves in some cases.1,4,6
NBTE tends to be clinically indolent until an advanced complication occurs, such as embolization or valvular dysfunction. Embolic events are common in patients with NBTE; nearly one-half of patients experience a systemic embolic event, often at the time of presentation. Cerebral, coronary, renal, and mesenteric circulations are most frequently involved, with manifestations that include focal neurologic deficits, vision changes, memory loss, pain in the extremities or flank, and acute abdomen. Patients present with valvular destruction or heart failure less frequently; murmurs are only appreciated in around one-quarter of cases.1,4-6 In addition to addressing the underlying cause, anticoagulation is the cornerstone of therapy for NBTE. Oral vitamin K antagonists (eg, warfarin) may not be as effective as low-molecular-weight heparin. Surgical intervention can be considered for some patients, such as those with severe valvular dysfunction or recurrent embolic events. Unlike patients with IE who typically undergo valve replacement, preservation of the affected valve is often possible in patients with NBTE.1
What are the causes of nonbacterial thrombotic endocarditis?
A 68-year-old man with an extensive smoking history presents with night sweats, weight loss, and hemoptysis. A 36-year-old woman with malar rash, generalized lymphadenopathy, and pancytopenia. Ironically categorized under Noninfective endocarditis.
Which malignancies are most often associated with nonbacterial thrombotic endocarditis? Which autoimmune conditions are associated with nonbacterial thrombotic endocarditis? What infections are associated with
Lung cancer.
Systemic lupus erythematosus (SLE).
Infection.
NBTE tends to develop in patients with advanced and metastatic malignancy, sometimes in association with disseminated intravascular coagulation. On autopsy, valvular distortion is minimal, suggesting that there is limited time between developing NBTE and death in cancer patients. Malignancies most frequently associated with NBTE include mucin-secreting adenocarcinomas of the lung, ovary, biliary system, pancreas, and stomach. Adenocarcinomas of the lung and ovary represent around one-half of cases.1,4
NBTE can be associated with SLE (including Libman-Sacks endocarditis), antiphospholipid syndrome (primary or secondary), rheumatic heart disease, and rheumatoid arthritis. These patients tend to have more significant valvulopathy (ie, regurgitant lesions) than patients with malignancy-associated NBTE because a comparatively longer clinical course allows for its development. Libman-Sacks endocarditis is a subtype of NBTE that occurs in patients with SLE, particularly in those with antiphospholipid syndrome. The vegetations of Libman-Sacks endocarditis are histologically distinct and tend to form on the ventricular aspect of the posterior mitral leaflet, although clinical manifestations are similar to other causes of NBTE.4,7 NBTE can be associated with acute infectious conditions, such as sepsis and pneumonia, and chronic infectious conditions, such as tuberculosis, osteomyelitis, and chronic pyelonephritis. It is important to note that the vegetations of
582
nonbacterial infectious NBTE are sterile.6 thrombotic endocarditis?
583
Infective Endocarditis
What is Infective endocarditis?
Which heart valves are most frequently involved in Infective endocarditis? What are the symptoms of Infective endocarditis? What are the physical findings of Infective endocarditis?
IE refers to the presence of endocardial vegetations composed primarily of platelets, fibrin, and microorganisms, most often involving the heart valves. Despite frequent episodes of transient bacteremia related to daily activities, such as chewing and tooth brushing, healthy endothelium is resistant to infection. However, damaged endothelium is susceptible to the direct adherence of microorganisms, leading to subsequent infection, or the development of microthrombi that later become infected. Endothelial damage may develop as a result of degenerative valvular lesions (eg, mitral valve prolapse), trauma from the impact of high-velocity blood jet due to turbulent blood flow (eg, aortic stenosis), or direct trauma from electrodes or catheters.3,8
IE most often involves the left-sided heart valves. This is thought to occur for 3 main reasons: (1) there is more turbulent blood flow on the left side of the heart as a result of higher pressures, predisposing the aortic and mitral valves to endothelial injury; (2) higher oxygen content of arterial blood is supportive of bacterial growth; and (3) a higher proportion of congenital and acquired predisposing heart conditions are left-sided. Intravenous drug users often develop endothelial injury involving the tricuspid valve as a result of repeated intravenous injections of solid particles, predisposing this population to right-sided endocarditis.3,9 Symptoms of IE may include chills, anorexia, weight loss, dyspnea, and flank pain.10
Physical findings of IE may include fever (in up to 90% of patients), heart murmur (in up to 85% of patients), signs of heart failure (eg, elevated JVP, gallop), splenomegaly, petechiae, splinter hemorrhages (Figure 29-2), Osler’s nodes (tender erythematous or violaceous subcutaneous nodules on the thenar and hypothenar eminences and pulps of fingers and toes), Janeway lesions (nontender erythematous or hemorrhagic macular lesions on the palms and soles) (Figure 29-3), Roth spots (retinal hemorrhages with a central pallor) on fundoscopic examination (Figure 29-4), and focal lung findings (in patients with right-sided IE). Systemic emboli may lead to other physical findings (eg, focal neurologic deficits).8,10
FIGURE 29-2 Splinter hemorrhage in a patient with Infective endocarditis. (From Stoller JK, Nielsen C, Buccola J, Brateanu A. The Cleveland Clinic Foundation Intensive Review of Internal Medicine. 6th ed. Philadelphia, PA: Wolters Kluwer Health; 2014.)
FIGURE 29-3 Janeway lesions in a patient with Infective endocarditis. (From Positano RG, DiGiovanni CW, Borer JS, Trepal MJ. Systemic Disease Manifestations in the Foot, Ankle, and Lower Extremity. Philadelphia, PA: Wolters Kluwer; 2017.)
What clinical manifestations are unique to right-sided
FIGURE 29-4 Roth spots (arrows) in a patient with Infective endocarditis. (Reprinted with permission from Hess RL. Roth spots in native valve.endocarditis. J Am Osteopath Assoc. 2013;113(11):863. doi:10.7556/jaoa.2013 063.)
In addition to the clinical manifestations shared with left-sided IE (eg, chills, fever, murmur), patients with right-sided IE may experience cough and pleuritic chest pain, and develop focal pulmonary findings on examination (eg, signs of consolidation or pleural effusion), which are related to cardiopulmonary septic emboli. In the absence of a right-to-left shunt, right-sided IE is not associated with systemic emboli or peripheral stigmata (eg, Janeway lesions). Signs of right-
584
Infective endocarditis? How is the diagnosis of Infective endocarditis established?
sided heart failure (eg, elevated JVP, peripheral edema) are more common in patients with right-sided IE.11 The diagnosis of IE can be definitively made with histologic and microbiologic examination of vegetations. However, a diagnostic tool, known as the Duke criteria, incorporates clinical, laboratory, and echocardiographic findings to identify the likelihood of IE (Table 29-1). The diagnosis is established with a specificity of 99% when any of the following are present: (1) 2 major criteria; (2) 1 major criterion and 3 minor criteria; (3) 5 minor criteria.
Table 29-1 Modified Duke Criteria for Infective Endocarditis12-14
Major Criteria 1. positive blood cultures for ie. A. two separate blood cultures positive for an organism typical for ie, or B. persistently positive blood cultures with an organism less typical for ie.a 2. evidence of endocardial involvement. A. Positive echocardiogram for IE. i. Oscillating intracardiac mass; or ii. Abscess; or iii. New partial dehiscence of prosthetic valve; or B. New valvular regurgitation by physical examination (ie, murmur consistent with valvular regurgitation).b
Minor Criteria 1. Predisposition: predisposing heart condition or IV drug use. 2. Fever: temperature ≥38.0°C (100.4°F). 2. Vascular phenomena: major arterial emboli, septic pulmonary infarcts, mycotic aneurysms, intracranial hemorrhage, conjunctival hemorrhages, and Janeway lesions. 3. Immunologic phenomena: glomerulonephritis, Osler’s nodes, Roth spots, and positive rheumatoid factor. 4. Microbiologic evidence: positive blood culture not meeting major criterion or serologic evidence of active infection with an organism consistent with IE.
aPersistently positive blood cultures is defined as recovery of a microorganism from at least 2 blood cultures drawn more than 12 hours apart, or all of 3, or a majority of 4 or more separate blood cultures (with first and last drawn at least 1 h apart). bAn isolated regurgitant lesion on echocardiography must be considered carefully, as mechanism is often difficult to determine and clinically insignificant valvular regurgitation is common in the general population.
How is In accordance with the Duke criteria (see Table 29-1), echocardiography can be diagnostic of IE when it identifies a echocardiography vegetation, perivalvular abscess, or new dehiscence of a prosthetic valve. It can be suggestive of IE if there is evidence useful in the of valvular destruction, prolapse, aneurysm, or perforation, or rupture of the chordae tendineae or papillary muscle. diagnosis of Isolated valvular regurgitation on echocardiography must be considered carefully, as mechanism is often difficult to Infective determine and clinically insignificant valvular regurgitation is common in the general population. Sequelae of endocarditis? endocarditis, such as valvular regurgitation and left ventricular dysfunction, can be monitored with echocardiography.15 What are the The goal of treatment for IE is eradication of infection. The antimicrobial agents of choice depend on the specific general principles organism(s) involved, however prolonged parenteral bactericidal therapy is generally necessary. It is recommended of therapy for that 3 sets of blood cultures be drawn 1 hour apart before the administration of antibiotics. The recovery rate of blood Infective cultures is reduced by up to 40% when antimicrobial agents are administered before blood culture collection. Surgical endocarditis? intervention may be necessary in some cases of IE, particularly when there are complications.2,10 What are the Heart failure is the most common complication of IE, occurring in over one-half of all cases. It typically occurs as a complications of result of valvular dysfunction, developing more frequently in patients with aortic valve involvement. Systemic Infective embolism occurs in up to one-half of patients with IE, most often involving the central nervous system, spleen, endocarditis? kidneys, lungs, and liver. Risk is highest earlier in the course of the disease (within 2 weeks of diagnosis), when vegetations are large (>10 mm), and when the mitral valve is involved. Risk decreases after initiation of antimicrobial therapy. Septic emboli may cause secondary infection in the blood vessel wall, resulting in the formation of a mycotic aneurysm. Intracardiac abscess develops in less than one-half of patients with IE but is more common in those with prosthetic valve endocarditis, and those with native aortic valve disease.3,8,16 What is the The prognosis of IE depends on many factors such as the infecting organism, site of infection, and clinical prognosis of circumstances including patient-related factors and the presence of complications. However, the overall in-hospital Infective mortality rate of IE approaches 25%; 5-year mortality is approximately 40%. Survivors should continue to be endocarditis? monitored for relapse or reinfection, and progressive valvulopathy.3,8 What are the 3 IE can be separated into the following subcategories: native valve endocarditis (NVE), prosthetic valve endocarditis general (PVE), and IE related to intravenous drug use (IVDU). subcategories of Infective endocarditis?
585
586
Native Valve Infective Endocarditis
How sensitive is echocardiography in identifying lesions that are diagnostic of native valve endocarditis? What are the 2 subtypes of native valve endocarditis based on clinical course?
What are the clinical differences between acute and subacute Infective endocarditis?
The sensitivity of transthoracic echocardiography (TTE) in identifying vegetations in NVE is approximately 75%; sensitivity improves to 90% with transesophageal echocardiography (TEE). The sensitivity of TTE in identifying a perivalvular abscess in NVE is approximately 50%; sensitivity improves to 90% with TEE.15
NVE can follow an acute or subacute clinical course.
Acute IE manifests suddenly (ie, within days) as a fulminant illness with rapid destruction of cardiac structures, hematogenous seeding of extracardiac sites, and progression to death within weeks if left untreated. In contrast, subacute IE tends to be more indolent with nonspecific symptoms (eg, night sweats, weight loss), causing slow damage to cardiac structures and rare extracardiac metastases. Subacute IE is associated with a more favorable prognosis. The classic clinical manifestations of IE (eg, Osler’s nodes) are more common in patients with subacute IE; such manifestations tend to be more limited in patients with acute IE, particularly right-sided disease in IV drug users.2,14
587
Acute Native Valve Infective Endocarditis
What are the causes of acute native valve endocarditis?
Gram-positive cocci in clusters. Gram-positive cocci in chains. A coagulase-negative species of Staphylococcus.
What is the treatment of choice for native valve endocarditis caused by Staphylococcus aureus? Which non– viridans streptococcal species cause acute native valve endocarditis? What is the prognosis of native valve endocarditis caused by Staphylococcus lugdunensis?
Staphylococcus aureus.
Non–viridans streptococci. Staphylococcus lugdunensis.
Staphylococcus aureus is the most common cause of acute NVE in the industrialized world. Resistance to oxacillin or methicillin (ie, methicillin-resistant Staphylococcus aureus [MRSA]) is on the rise, even in patients without risk factors (eg, health care exposure). Staphylococcus aureus IE in non-IV drug users typically involves the left-side of the heart (eg, aortic and mitral valves). The treatment of choice for uncomplicated methicillin-sensitive Staphylococcus aureus (MSSA) NVE is intravenous nafcillin or oxacillin for 6 weeks. For uncomplicated MRSA NVE, the agent of choice is vancomycin.2
Streptococcal species are a frequent cause of IE; however, most cases are caused by viridans streptococci and are subacute in presentation. Non–viridans streptococcal species are capable of causing acute IE. These species include groups A, B, C, F, and G streptococci, and Streptococcus pneumoniae. Alcoholism is the most common risk factor for Streptococcus pneumoniae IE, which is present in nearly one-third of cases. Pneumonia and meningitis may also be present, forming the classic triad. The treatment of choice for non–viridans streptococcal NVE is penicillin or ceftriaxone for 4 to 6 weeks. Gentamycin may be added for the first 2 weeks in patients with infection caused by serogroups B, C, F, and G.2,17 Traditionally associated with PVE, coagulase-negative staphylococci are emerging as important causes of NVE, particularly in the health care setting. In contrast with the subacute course characteristic of NVE caused by other coagulase-negative staphylococci, Staphylococcus lugdunensis IE tends to be particularly aggressive and is associated with a high rate of perivalvular abscess formation, peripheral emboli, and mortality (similar to Staphylococcus aureus IE). Patients should be treated with standard regimens based on in vitro susceptibility patterns.8,18
588
Subacute Native Valve Infective Endocarditis
What are the causes of subacute native valve endocarditis?
Part of normal oral flora, this group of organisms has historically caused most cases of subacute NVE. Once classified as a streptococcal species. IE caused by this organism often occurs in patients with abnormalities of the colon. Traditionally a source of PVE, this organism is becoming a more common cause of subacute NVE. A group of organisms that require additional time to grow in the laboratory. Culture may not reveal an organism in up to 10% of cases of IE.
Which of the viridans streptococci are most commonly associated with native valve endocarditis? What are the typical sources of enterococcal endocarditis?
Which diagnostic procedure should be
Viridans streptococci.
Enterococci.
Streptococcus gallolyticus (formerly Streptococcus bovis).
Staphylococcus epidermidis.
HACEK organisms: Haemophilus species, Aggregatibacter species, Cardiobacterium hominis, Eikenella corrodens, and Kingella species.
Culture-negative IE.2
Viridans streptococci are commensals of the oral, Gastrointestinal, and urogenital tract. The species that most often cause subacute NVE are Streptococcus sanguis, Streptococcus mitis, Streptococcus salivarius, and Streptococcus mutans. Most strains are highly susceptible to penicillin, and cure rates approach 100% in uncomplicated cases after 4 weeks of IV therapy.2,8
Enterococci account for around 10% of all cases of IE. The vast majority of cases are caused by Enterococcus faecalis, with the remainder caused by Enterococcus faecium. Sources of these organisms include the Gastrointestinal and genitourinary tracts. Susceptibility testing is important in cases of enterococcal IE. In susceptible strains, optimal therapy for enterococcal NVE includes penicillin, ampicillin, or vancomycin in combination with either gentamicin or streptomycin for 4 to 6 weeks. Alternative regimens may be necessary in patients with resistant strains or impaired renal function (eg, ampicillin plus ceftriaxone). Enterococcal IE may have an acute presentation.2 In patients with Streptococcus gallolyticus endocarditis, colonoscopy should be performed to evaluate for ulcerative lesions of the colon caused by underlying conditions such malignancy or inflammatory bowel disease. Like viridans streptococci, most strains of Streptococcus gallolyticus are highly susceptible to penicillin, and cure rates approach 100% in uncomplicated cases after 4 weeks of IV therapy.2
589
considered for patients with Streptococcus gallolyticus endocarditis? What are the Coagulase-negative staphylococci are part of the normal skin flora. These organisms are responsible for an increasing features of proportion of cases of NVE, particularly Staphylococcus epidermidis. Extensive contact with health care is an important risk Staphylococcus factor. Despite the subacute presentation, coagulase-negative staphylococci NVE is associated with high rates of epidermidis perivalvular abscess formation, heart failure, and death. Most strains are methicillin resistant and must be treated similar native valve to MRSA (eg, vancomycin).2,8,19 endocarditis? How common The HACEK organisms account for up to 10% of cases of community-acquired NVE in patients who do not use IV drugs. is community- Growth of these organisms is slow in standard blood culture media, and identification requires prolonged incubation. acquired native The treatment of choice is ceftriaxone for 4 weeks.2 valve endocarditis related to HACEK organisms? What is Culture-negative endocarditis refers to IE in which an organism is not identified by blood culture. It may occur for a culture- variety of reasons, including inadequate microbiological techniques, infection with fastidious or noncultivable negative organisms, or administration of antimicrobial agents before collecting blood cultures. In some cases, blood culture yield endocarditis? may return after several days without antibiotics. Organisms that are associated with negative blood cultures include Streptococcus species in which antibiotics have been administered before culture acquisition, Abiotrophia and Granulicatella species (formerly known as nutritionally variant streptococci), Coxiella burnetii, Bartonella species, Brucella species, Legionella species, Tropheryma whipplei, and fungi. Other microbiological techniques (eg, serologies) may be helpful. Empiric antimicrobial therapy to cover all likely pathogens is necessary in patients with culture-negative endocarditis.2
590
Prosthetic Valve Infective Endocarditis
How common is prosthetic valve endocarditis? Which type of prosthetic valve (mechanical or bioprosthetic) is more likely to become infected? How sensitive is echocardiography in identifying lesions that are diagnostic of prosthetic valve endocarditis? What are the 2 subtypes of prosthetic valve endocarditis based on the timing of onset?
What time course is generally used to define early versus late prosthetic valve endocarditis? What are the general microbiological differences between early and late prosthetic valve endocarditis?
PVE accounts for up to one-third of all cases of IE. It is associated with a higher mortality rate than NVE.20 In the first year after surgery, mechanical valves are more likely than bioprosthetic valves to become infected, but at 5 years, there is no difference in risk.21
In patients with PVE, the sensitivity of TTE in identifying vegetations, a perivalvular abscess, or dehiscence of the prosthetic valve is around 25%; sensitivity improves to 90% with TEE.15,22
PVE can occur in the early or late period following valve replacement surgery.
PVE is considered early when it occurs within 2 months of valve replacement surgery (less common); late PVE occurs >12 months after surgery (more common). This dichotomy is helpful because the organisms associated with each subtype are distinctive. When PVE occurs between 2 and 12 months after surgery, it is referred to as intermediate and can be caused by organisms associated with both early and late PVE.14,20 The organisms of early PVE tend to be nosocomial in nature, whereas the organisms of late PVE tend to mirror the community-acquired organisms found in patients with NVE.
591
Early Prosthetic Valve Infective Endocarditis
What are the causes of early prosthetic valve endocarditis?
PVE caused by this organism has recently been on the rise and is now considered the most common cause of early PVE. Traditionally, this organism had been considered the most common cause of early PVE. PVE caused by these organisms may be associated with infection of the genitourinary or non–oral Gastrointestinal tract. Nonbacterial organisms. Commonly assumed to be a contaminant when isolated from blood cultures.
What is the optimal antibiotic therapy for prosthetic valve endocarditis caused by Staphylococcus aureus? What are the features of prosthetic valve endocarditis caused by Staphylococcus epidermidis? Which non-HACEK gram-negative bacilli are associated with prosthetic valve endocarditis? Which fungus is most frequently associated with prosthetic valve
Staphylococcus aureus.20
Staphylococcus epidermidis.
Non-HACEK gram-negative bacilli.
Fungi. Diphtheroids (ie, Corynebacterium species).
PVE caused by Staphylococcus aureus is associated with a high mortality rate, and combination antibiotics are recommended. For MSSA PVE, triple-drug therapy is preferred, including nafcillin or oxacillin plus rifampin for at least 6 weeks with gentamycin for the first 2 weeks. For MRSA PVE, the synthetic penicillin should be replaced with vancomycin.2
Coagulase-negative staphylococcus PVE is almost always caused by Staphylococcus epidermidis. It tends to be aggressive with an acute presentation similar to that of Staphylococcus aureus, with high rates of complications such as perivalvular abscess, heart failure, and peripheral metastases.20,21
Among the non-HACEK gram-negative bacilli, Escherichia coli and Pseudomonas aeruginosa are most frequently associated with PVE. Others include Klebsiella species, Serratia species, and Proteus species. Most cases are associated with health care exposure, and organisms are frequently resistant to multiple antibiotics. The combination of a β-lactam agent (eg, penicillin, cephalosporin, carbapenem) with either an aminoglycoside or fluoroquinolone for 6 weeks is a reasonable option. Despite combined medical and surgical approaches, in-hospital mortality rates are high (approximately 25%).2,23 Candida species cause most cases of fungal PVE. Complications such as systemic embolization and perivalvular abscess occur frequently. Optimal antimicrobial treatment consists of amphotericin B with or without flucytosine for at least 6 weeks followed by chronic suppression with fluconazole. Combination medical and surgical therapy is required in most cases; however, despite treatment, more than one-half of patients die. Candida is also associated with
592
endocarditis? What are the features of prosthetic valve endocarditis caused by diphtheroids?
late PVE.24 The diphtheroids are part of normal skin flora and are frequent contaminants of blood cultures. However, these organisms cause up to 10% of early PVE cases and just fewer than 5% of late PVE cases. The isolation of diphtheroids in blood cultures often requires prolonged incubation, but the Gram stain can be revealing (gram-positive bacilli). Complications such as perivalvular abscess, valvular dysfunction, and heart failure are common. Based on susceptibility data, penicillin in combination with gentamycin, or vancomycin alone may be used. A significant proportion of cases require the combination of medical and surgical treatment. Despite therapy, mortality rates are high at about 40%.25,26
593
Late Prosthetic Valve Infective Endocarditis
What are the causes of late prosthetic valve endocarditis?
These organisms, which are part of normal oral flora, are a leading cause of subacute NVE. The most common cause of acute NVE. A common cause of early PVE; this organism has a predilection for hardware. Formerly classified as group D Streptococcus. Streptococcus pyogenes belongs to this group of organisms.
What is the treatment for prosthetic valve endocarditis caused by viridans streptococci? How often does Staphylococcus aureus cause late prosthetic valve endocarditis? What is the optimal antibiotic therapy for prosthetic valve endocarditis caused by Staphylococcus epidermidis? What is the optimal antibiotic therapy for patients with prosthetic valve endocarditis caused by Enterococcus species? How often do non–viridans streptococci cause late prosthetic valve endocarditis?
Viridans streptococci.
Staphylococcus aureus. Staphylococcus epidermidis.
Enterococcus. Non–viridans streptococci (groups A, B, C, F, and G streptococci, and Streptococcus pneumoniae).
Viridans streptococcal PVE is almost always late-onset. Treatment of choice includes penicillin or ceftriaxone for 6 weeks with or without gentamycin for the first 2 weeks; penicillin-resistant strains should be treated with penicillin or ceftriaxone plus gentamycin for 6 weeks.2,20 Staphylococcus aureus frequently causes both early and late PVE; in some populations, it is the most common cause of both. It is a virulent organism with a high rate of complications such as perivalvular abscess, purulent pericarditis, and peripheral metastases.20 The antimicrobial regimen used to treat PVE caused by Staphylococcus epidermidis is the same as for PVE caused by Staphylococcus aureus. For methicillin-sensitive strains, triple-drug therapy is preferred, including nafcillin or oxacillin plus rifampin for at least 6 weeks with gentamycin for the first 2 weeks. For methicillin- resistant strains, the synthetic penicillin should be replaced with vancomycin.2
Enterococcus species cause approximately 10% of cases of late PVE. Medical treatment for enterococcal PVE and NVE is the same: in susceptible strains, optimal therapy includes penicillin, ampicillin, or vancomycin in combination with either gentamicin or streptomycin. Duration of treatment should be extended to at least 6 weeks. Alternative regimens may be necessary in patients with resistant strains or impaired renal function.2 Non–viridans streptococci cause approximately 10% of late PVE. Medical treatment for non–viridans streptococcus PVE and NVE is the same, although the duration of therapy for PVE is Increased to at least 6 weeks.2,20
Similar to subacute NVE, HACEK organisms and culture-negative endocarditis can also cause late PVE.
594
595
Infective Endocarditis Related to Intravenous Drug Use
Which heart valves are most frequently infected in intravenous drug users? What physical findings are common in patients with right-sided endocarditis from intravenous drug use? What are the manifestations of embolic events in patients with right-sided endocarditis?
Native or prosthetic valves may be infected in IV drug users. These patients often develop endothelial injury involving the tricuspid valve as a result of repeated IV injections of solid particles, resulting in a predisposition for right-sided endocarditis. The affected valve is the tricuspid in 50% of cases; the mitral and aortic valves in 20% of cases each, and multiple valves are involved in 10% of cases. The pulmonic valve is rarely involved. The type of IV drug used may influence the site that is affected (eg, tricuspid valve disease is more frequent in heroin users compared with those who use other IV drugs).9,11,27,28 Fever is common in IE involving either side of the heart. Evidence of right-sided heart failure (eg, elevated JVP, peripheral edema) caused by tricuspid regurgitation is common in patients with endocarditis related to IV drug use. Tricuspid regurgitation may be associated with a holosystolic murmur best heard over the left lower sternal border that may augment with inspiration (Carvallo’s sign). There may also be CV fusion in the jugular venous waveform, which is known as Lancisi’s sign. For videos of Carvallo’s sign and Lancisi’s sign, see the associated references.11,29,30
Embolic events in patients with right-sided endocarditis typically affect the lungs, manifesting as multiple peripheral- based consolidations that are often cavitary on imaging (Figure 29-5). These patients may complain of dyspnea, chest pain, and cough. In patients with right-to-left shunt, systemic embolic phenomena can occur.31
FIGURE 29-5 Multiple septic pulmonary emboli demonstrated on chest imaging in a patient with right-sided Infective
What is the optimal treatment for endocarditis related to intravenous drug use?
endocarditis. Note the presence of cavitation in some of the lesions. (From Muller NL, Franquet T, Lee KS, Silva CIS. Imaging of Pulmonary Infections. Philadelphia, PA: Lippincott Williams & Wilkins; 2007.)
The antimicrobial treatment for IE among IV drug users depends on the organism(s) and site of involvement. Duration of therapy is 4 to 6 weeks in most cases, however in select groups a 2-week treatment course may be effective. Surgical intervention may be beneficial in some patients, such as those with refractory heart failure, persistent infection, perivalvular abscess, conduction abnormalities, or prosthetic valve involvement. However, surgical intervention in these patients must be considered carefully given the significant risk of recidivism. Right- sided IE is associated with a favorable prognosis (with mortality <10%).31,32
What are the causes of Infective endocarditis related to intravenous drug use?
This organism is responsible for more than one-half of cases of IE in IV drug users. Streptococcus mitis is an organism within this group. This genus contains over 15 species, but only 2 are commonly associated with endocarditis. May be seen in patients who contaminate needles, inoculate injection sites with saliva (via skin or needle licking), or use their teeth to crush tablets before injection. Infection with this group of organisms is often considered a “stand-alone” indication for
Staphylococcus aureus.14
Viridans streptococci. Enterococcus.2
Eikenella corrodens (a HACEK organism).31
Fungi.2
596
surgery.
Which heart valves are most frequently involved in intravenous drug users with Staphylococcus aureus Infective endocarditis? Which heart valves are most frequently infected in intravenous drug users with viridans streptococcal Infective endocarditis? Which heart valves are most frequently infected in intravenous drug users with enterococcal Infective endocarditis? Which gram-negative bacilli are associated with Infective endocarditis among intravenous drug users?
Which fungi are associated with Infective endocarditis among intravenous drug users?
Most cases of Staphylococcus aureus IE in IV drug users involve the tricuspid valve. Cases are frequently complicated by extracardiac infection, such as pneumonia, soft tissue abscess, septic arthritis, and osteomyelitis. Nafcillin or oxacillin is the agent of choice for MSSA, whereas vancomycin is the agent of choice for MRSA.14,31-33 The vast majority of cases of viridans streptococcal IE in IV drug users are left-sided. Penicillin is the agent of choice.14,31,32
The vast majority of cases of enterococcal IE in IV drug users are left-sided.14,31
IV drug use is a risk factor for IE caused by gram-negative bacilli, most frequently Pseudomonas aeruginosa and Escherichia coli. Others include the HACEK organisms, Serratia marcescens, and Proteus mirabilis. The use of tap water or toilet water in drug preparation is a risk factor for infection with Pseudomonas aeruginosa. Gram-negative IE among IV drug users is associated with a less favorable prognosis compared with IE caused by more typical organisms in this population.2,32,34 Candida and Aspergillus species cause most cases of fungal endocarditis among IV drug users. Candida albicans is far more common, and is associated with positive blood cultures; Aspergillus endocarditis rarely produces positive blood cultures. Treatment almost always requires the combination of parenteral antifungal therapy and surgery. Fungal IE among IV drug users is associated with a less favorable prognosis compared with IE caused by more typical organisms in this population.2,13
Polymicrobial IE is more common among IV drug users, typically involves the tricuspid valve, and is associated with a poor prognosis. 31
597 Case Summary feA 31-year-old man with a history of IVDU presents with acute-onset ver and dyspnea and is found to have elevated JVP, several heart murmurs, and a variety of other abnormal physical findings.
What is the most likely underlying diagnosis in this patient? Infective endocarditis.
598
Bonus Questions
Which general This case describes acute left-sided NVE in an IV drug user. Categorizing endocarditis can be helpful in determining the most likely underlying organisms and Infective categories of guiding treatment.
endocarditis apply to this case? What clinicals This case meets 1 major Duke criterion (new valvular regurgitation by physical examination) and 3 minor criteria (predisposition [IVDU], fever, and immunologic suggestive of case are findings in thi phenomena) (see Table 29-1). Infective endocarditis? significance of in shape, and is best heard over the third intercostaltspace of the left sternal border (Erb’s point). There is often antassociated systolic ejection murmur that occurs asto What is the The heart murmurs described in this case are consis ent with aortic regurgitation. The murmur of aortic regurgita ion typically begins early in diastole, is decrescend the heart the regurgitant bolus of blood generates turbulence on its way back through the aortic valve (technically a flow murmur). Aortic regurgitation may be associated wi h murmurs in this a low-pitched, “blubbering,” mid-to-late diastolic murmur heard over the apex, known as the Austin Flint murmur. The mechanism is thought to be distortion and case? early closure of the anterior leaflet of the mitral valve caused by the regurgitant aortic jet, resulting in functional mitral stenosis. For a video demonstrating the mechanism of the Austin Flint murmur, see the associated reference.35,36 What is the Nail bed pulsation, known as Quincke’s pulse, is a sign of aortic regurgitation. Regurgitant blood flow into a dilated left ventricle during diastole leads to a decrease significance of in diastolic pressure and a consequent increase in stroke volume, resulting in blanching and flushing, respectively, of the nail bed. For a video of Quincke’s pulse, see the the nail bed . pulsation in this associated reference 37 case? What is the of The to-and-fro head bob, known as de Musset sign, is a sign of aortic regurgitation. Regurgitant blood flow into the left ventricle during diastole leads to an increase the head bob in significance in stroke volume, the force of which causes the head to jolt back with each heartbeat. 35 this case? What is the of The painful erythematous nodules demonstrated in Figure 29-1, known as Osler’s nodes, are immunologic phenomena that occur in patients with IE. Peripheral demonstrated in the skin finding significance manifestations such as these are more common in patients with subacute IE but can occur in acute cases (such as this one). 2 Figure 29-1? Which organis IE related to IVDU is most often caused by Staphylococcus aureus, viridans streptococci, and Enterococcus infections. Empiric coverage of these organisms while are most oftenms awaiting blood culture results would be a reasonable approachiin this case. Thelpossiblity of infection with gram-negative bacilli or fungi should always be considered endocarditis Infective associated with in IV drug users. Given the evidence of severe aortic regurgitat on and heart fai ure, this patient should also be evaluated for aortic valve replacement. 16 related to intravenous drug use?
599 Key Points
Endocarditis refers to inflammation of the endocardium, usually Endocarditis can be Noninfective or Infective. involving the heart valves. Nonbacterial thrombotic endocarditis refers to the presence of sterile vegetations on the endocardium that develop in association NBTE most often presents with embolic phenomena. with a variety of inflammatory or hypercoagulable conditions. Anticoagulation is the cornerstone of medical management. Infective endocarditis is a clinical diagnosisibased on history, physical examination, laboratory data, and maging (echocardiography).
IE can be separated into the following subcategories: NVE, PVE, NVE can follow an acute or subacute clinical course. and IE related to IV drug use. Staphylococcus aureus is the most common cause of acute NVE. Viridans streptococci is the most common cause of subacute NVE. PVE can occur in the early period (<2 months) or late period (>12 months) following valve replacement surgery.
Early PVE is most commonly caused by Staphylococcus aureus and IE related to IVDU is mostlcommonly caused by Staphylococcus The tricuspid valve is most frequently involved in IE associated Staphylococcus epidermidis. Late PVE is most common y caused by viridans streptococci. aureus. with IVDU, but left-sided involvement is also common.
600
References 1. Aryana A, Esterbrooks DJ, Morris PC.tNonbacterial thrombotic endocarditis with. recurrent embolic events as manifesta ion of ovarian neoplasm. J Gen Intern Med 2006;21(12):C12-C15.
- Baddour LM, Wilson WR, Bayer AS, et al. Infective endocarditis in adults: diagnosis, healthcare professionals from the American Heart Association. Circulation. antimicrobial therapy, and management of complications:a scientific statement for 2015;132(15):1435-1486.
-
Hoen B, Duval X. Clinical practice. Infective endocarditis. N Engl J Med. 2013;368(15):1425-1433. Eiken PW, Edwards WD, Tazelaar HD, McBane RD, Zehr KJ. Surgical pathology
-
nonbacterial thrombotic endocarditis in 30 patients, 1985-2000. Mayo Clin Proc. of 2001;76(12):1204-1212.
-
Liu J, Frishman WH. Nonbacterial thrombotic-endocarditis: pathogenesis, diagnosis, 6. Llenas-Garcia J, Guerra-Vales JM, Montes-Moreno S, Lopez-Rios F, Castelbon- and management. Cardiol Rev. 2016;24(5):244 247. Fernandez FJ, Chimeno-Garcia J. Nonbacterial thrombotic endocarditis:
clinicopathologic study of a necropsy series. Rev Esp Cardiol. 2007;60(5):493-500. 7. Moyssakis I, Tektonidou MG, Vasilliou VA, Samarkos M, Votteas V, Moutsopoulos HM. Libman-Sacks endocarditis in systemic lupus erythematosus: prevalence,
associations, and evolution. Am J Med. 2007;120(7):636-642. 8. Cahill TJ, Prendergast BD. Infective endocarditis. Lancet. 2016;387(10021):882-893. 9. Frontera JA, Gradon JD. Right-side endocarditis in injection drug users: review of proposed mechanisms of pathogenesis. Clin Infect Dis. 2000;30(2):374-379.
-
Beynon RP, Bahl VK, Prendergast BD. Infective endocarditis. BMJ. 2006;333(7563):334- 339.
-
Ashley E, Niebauer J. Cardiology Explained. London: Remedica; 2004.
-
Durack DT, LukesiAS, Bright DK. New criteria for diagnosis of Infective endocarditis: utilization of specif c echocardiographic findings. Duke Endocarditis Service. Am J Med. 1994;96(3):200-209.
-
Li JS, Sexton DJ, Mick N, et al. Proposed modifications to the Duke criteria for the diagnosis of Infective endocarditis. Clin Infect Dis. 2000;30(4):633-638.
-
Longo DL, Fauci AS, Kasper DL, Hauser SL, Jameson JL, LoscalzolJ, eds. Harrison’s 15echocardiography in Infective endocarditis. Eur J Echocardiogr. 2010;11(2):202-219. 16. McDonald JR. Acute Infective endocarditis. Infect Dis Clin North Am. 2009;23(3):643- Principles of Internal Medicine. 18th ed. New York, NY: McGraw-Hil ; 2012. . Habib G, Badano L, Tribouilloy C, et al. Recommendations for the practice of 664.
-
Aronin SI, Mukherjee SK, West JC, Cooney EL. Review of pneumococcal endocarditis in adults in the penicillin era. Clin Infect Dis. 1998;26(1):165-171. . Anguera I, Del Rio A, Miro JM, et al. Staphylococcus lugdunens s Infective
18endocarditis: description of 10 cases and analysis of.native valve,iprosthetic valve, and 19. Chu VH, Woods CW, Miro JM, et al. Emergence of coagulase-negative staphylococci pacemaker lead endocarditis clinical profiles. Heart 2005;91(2):e10. as a cause of native valve endocarditis. Clin Infect Dis. 2008;46(2):232-242.
- Lee JH, Burner KD, Fealey ME, et al. Prosthetic valve endocarditis: clinicopathological
601
correlates in 122 surgical specimens from 116 patients (1985-2004). Cardiovasc Pathol. 2011;20(1):26-35.
-
Bashore TM, Cabell C, Fowler V Jr. Update on Infective endocarditis. Curr Probl Cardiol. 2006;31(4):274-352.
-
Morguet AJ, Werner GS, Andreas S, Kreuzer H. Diagnostic value of transesophageal compared with transthoracic echocardiography in suspected prosthetic valve
endocarditis. Herz. 1995;20(6):390-398. 23. Morpeth S, Murdoch D, Cabell CH, et al. Non-HACEK gram-negative bacillus endocarditis. Ann Intern Med. 2007;147(12):829-835. . Boland JM, Chung HH, Robberts FJ, et al. Fungal prosthetic valve endocarditis: M
24Clinic experience with a clinicopathological analysis. Mycoses. 2011;54(4):354-360. ayo 25. Belmares J, Detterline S, Pak JB, Parada JP. Corynebacterium endocarditis species- specific risk factors and outcomes. BMC Infect Dis. 2007;7:4. . Murray BE, Karchmer AW, Moellering RC Jr. Diphtheroid prosthetic valve
26endocarditis. A study of clinical features and infecting organisms. Am J Med. 1980;69(6):838-848. . Mathew J, Addai T, Anand A, Morrobel A, Maheshwari P Freels S. Clinical feature
27site of involvement, bacteriologic findings, and;outcome of,Infective endocarditis in s, 28intravenous drug abusers:ian update. Eur J ClinlMicrobiol Infect Dis. 2012;31(11):2905- intravenous drug users. Arch Intern Med. 1995 155(15):1641-1648. . Sousa C, Botelho C, Rodr gues D, Azeredo J, O iveira R. Infective endocarditis in 2910.
- Burgess TE, Mansoor AM. Giant a waves. BMJ Case Rep. 2017;2017.
-
Mansoor AM, Mansoor SE. Images in clinical medicine. Lancisi’s sign. N Engl J Med. 2016;374(2):e2.
-
Colville T, Sharma V, Albouaini K. Infective endocarditis in intravenous drug users: a review article. Postgrad Med J. 2016;92(1084):105-111.
-
Ji Y,iKujtan L, Kershner D. Acute endocarditis in intravenous drug users: a case report 33. Ortiz-Bautista C, Lopez J, Garcia-Granja PE, et al. Current profile of Infective and l terature review. J Community Hosp Intern Med Perspect. 2012;2(1). endocarditis in intravenous drug users: the prognostic relevance of the valves
involved. Int J Cardiol. 2015;187:472-474. 34. Kaushik KS, Kapila K, Praharaj AK. Shooting up: the interface of microbial infections and drug abuse. J Med Microbiol. 2011;60(Pt 4):408-422.
- Marriott HJL. Bedside Cardiac Diagnosis. Philadelphia, PA: Lippincott; 1993.
-
Weir RA, Dargie HJ. Images in clinical medicine. Austin flint murmur. N Engl J Med. 2008;359(10):e11.
-
Mansoor AM, Mansoor SE. Images in clinical medicine. Quincke’s pulse. N Engl J Med. 2013;369(7):e8.
602
CHAPTER 30
603
Meningitis
604 Case: A 34-year-old man with agitated delirium complicated by end-stage renal disease, status posttkidneyitus A 34-year-old man with a history of type 1 diabe es mell transplantation from an unrelated donor, is admitted to the hospital
with several hours of headache, confusion, and agitation. Medications patient is agitated and disoriented. Neck pain and stiffness are . The puncture (LP) is performed, and the cerebrospinal fluid (CSF) appears subsequent sampling; vials 1 (left) and 4 (right) are identical in include prednisone, tacrolimus, and mycophenolate mofetil. Temperature is 37.8°C, and heart rate is 112 beats per minute present. Cross-sectional imaging of the head is unremarkable. Lumbar reddish in color. The intensity of the color remains consistent with appearance (Figure 30-1).
FIGURE 30-1 (Courtesy of Avital O’Glasser, MD, Carlton Scharman, MD, and Danielle M. Taylor.)
CSF studies are shown below: Appearance Opening Pressure (cm H2O) WBC/µL Glucose (mg/dL) Total Protein (mg/dL) Bloody 14 512 (92% lymphocytes) 63 88 abMagnetic resonance imaging (MRI) of the brain demonstrates normal signal involving the left medial temporal lobe. What is the most likely diagnosis in this patient?
What is meningitis? Meningitis refers to inflammation of the meninges associated with an abnormal increase in the number of white blood cells (WBCs) in the CSF (known as pleocytosis).1 What are the Depending on the underlying cause, meningitis can present as an acute fulminant illness with rapid progression symptoms of within hours, a subacute illness progressing over a period of days, or a chronic illness progressing over months. meningitis? The spectrum of clinical manifestations depends on the underlying cause, but symptoms may include headache, neck stiffness, neck pain, lethargy, nausea, vomiting, photophobia, phonophobia, and confusion (which is usually not prominent in the initial course of the illness).2 What are the physical Physical findings of meningitis depend on the underlying cause, but may include fever, nuchal rigidity, positive findings of Kernig’s sign (pain and resistance on extension of the knee starting with the hip and knee flexed at 90°), positive meningitis? Brudziński’s sign (reflex flexion of the hips and knees on passive flexion of the neck with the patient in the supine position), papilledema, and other signs of Increased intracranial pressure (eg, sixth nerve palsy). Some physical findings may be specific to underlying organisms (eg, the maculopapular and petechial skin rash of meningococcemia).2,3 What is the difference Encephalitis is inflammation of the brain parenchyma—usually caused by a viral infection—and is clinically between meningitis associated with neurologic dysfunction early in the course of the illness. Pleocytosis is often present in patients and encephalitis? with encephalitis but may be absent in some cases.4 What is The term meningoencephalitis refers to cases in which there are clinical features of both meningitis and meningoencephalitis? encephalitis; viral infection is a frequent cause.1 What are the 4 general Meningitis can be viral, bacterial, fungal, or aseptic. categories of
605
meningitis?
Historically, viral meningitis has been used synonymously with aseptic meningitis (at a time when viruses were not easily identified in the laboratory). These categories will be considered separate in this chapter.
What diagnostic procedure should be performed in patients with suspected meningitis? Which patients should undergo neuroimaging before lumbar puncture?
Should empiric antimicrobial therapy be withheld until a cerebrospinal fluid sample is obtained in patients with suspected meningitis? Which of the profiles in Table 30-1 are characteristic of normal cerebrospinal fluid, viral meningitis, typical bacterial meningitis, atypical bacterial meningitis, fungal meningitis, and aseptic meningitis?
LP should be performed urgently in most patients with suspected meningitis. However, LP can result in cerebral herniation in patients with Increased intracranial pressure caused by a mass lesion. In some patients, neuroimaging (eg, computed tomography [CT]) should be performed before LP to evaluate for mass lesions. CT imaging of the head should be performed before LP in patients who are at Increased risk for a mass lesion with associated Increased intracranial pressure. Such patients include those with immunocompromised status, history of central nervous system disease (eg, mass lesion), new-onset seizure (within 1 week of presentation), papilledema, abnormal level of consciousness, or focal neurologic deficits.5 When possible, LP should be performed before antimicrobial therapy is initiated, as this ensures optimal yield of CSF cultures. However, if there is an anticipated delay in performing LP, empiric therapy should be administered first.6 Fluid type A corresponds to normal CSF; type B to typical bacterial meningitis; type C to fungal meningitis (and some types of atypical bacterial meningitis); and type D to viral meningitis, most types of atypical bacterial meningitis, and most types of aseptic meningitis. Fluid types C and D may be characterized by a predominance of neutrophils early in the course of the associated illness. It is important to note that the numbers provided in Table 30-1 represent general rules of thumb.7
Table 30-1 Fluid type A B C D
WBC/µL <5 500-20,000 25-500 5-1000
Predominant WBC Lymphocytes Neutrophils Lymphocytes or neutrophils Lymphocytes
Glucose (mg/dL) Normal Low Low or normal Normal
Protein (mg/dL) <50 100-700 50-500 <100
How can it be determined if there is true pleocytosis in the context of a traumatic lumbar puncture?
After a traumatic LP, peripheral blood contamination of CSF will artificially increase the WBC count. Comparing the ratios of WBCs to red blood cells (RBCs) in both the peripheral blood and CSF can determine if there is true pleocytosis in the context of a traumatic LP. In a purely traumatic tap, the CSF RBC to WBC ratio should be similar to the peripheral RBC to WBC ratio. If the measured CSF WBC count exceeds that predicted by the peripheral ratio, then there were preexisting WBCs in the CSF, independent of the traumatic tap. For example:
Peripheral RBC (per µL) 4,000,000
Peripheral WBC (per µL) 10,000
Peripheral RBC:WBC Ratio 400:1
CSF RBC (per µL) 20,000
Expected CSF WBC (per µL) ≤50
If the measured CSF WBC count exceeds 50 cells/µL, then there were preexisting WBCs in the CSF (ie, true pleocytosis).7
606
Viral Meningitis
What is the typical opening pressure of viral meningitis? What is the characteristic cerebrospinal fluid profile of viral meningitis? Viral
The opening pressure of viral meningitis is typically normal (<18 cm H2O) but can be mildly elevated (up to 35 cm H2O) in some cases.7 Fluid type7 WBC (per µL) WBC Type Glucose (mg/dL) Protein (mg/dL) 5-1000 Lymphocytes Normal <100
Is viral meningitis always associated with a lymphocytic pleocytosis? How is the diagnosis of viral meningitis definitively made? What is the treatment for viral meningitis?
Neutrophils can predominate early in the course of viral meningitis, with a shift to lymphocytes after 12 to 48 hours.7 Polymerase chain reaction (PCR) of CSF can be used to diagnose specific causes of viral meningitis. PCR is 1000-fold more sensitive than routine viral culture. Definitive diagnosis of viral meningitis reduces unnecessary antibiotic exposure and decreases length of hospital stay.1 In general, viral meningitis is treated supportively and is usually self-limited. In contrast, viral encephalitis and meningoencephalitis can be life-threatening and antiviral treatment may be life-saving in some cases.1
What are the causes of viral meningitis?
This group of viruses includes coxsackieviruses A and B, poliovirus, and echovirus and most commonly causes meningitis during the summer and fall seasons in temperate climates. Infection with these viruses may result in recurrent lesions of the oral and genital regions. A peripheral protein gap can be a clue to infection with this virus. Excreted in the urine and feces of rodents. Arthropod-borne viruses. Parotitis and meningismus in an unvaccinated patient.
How common is enterovirus meningitis?
Which type of herpes simplex virus is responsible for most cases of viral meningitis in immunocompetent adults?
Enteroviruses.
Herpes simplex virus (HSV) types 1 (HSV-1) and 2 (HSV-2).
Human immunodeficiency virus (HIV).
Lymphocytic choriomeningitis virus (LCMV). Arboviruses. Mumps.
Enteroviruses are the most common cause of viral meningitis, accounting for most cases across all age groups. Children are most frequently affected. In temperate climates, outbreaks tend to be seasonal, occurring more often in the summer and autumn months. In tropical and subtropical climates, infection rates are high year-round. Most infections are asymptomatic, but a variety of neurologic manifestations can develop, including meningitis, meningoencephalitis, and paralytic poliomyelitis. Meningitis may be associated with mucocutaneous manifestations, such as hand, foot, and mouth disease in patients with coxsackie virus infection. Patients frequently experience moderate to high fever and several days of severe headache. Infection is typically self- limited with good prognosis.1,8 HSV is the second most common cause of viral meningitis. HSV-2 is responsible for most cases of HSV-related viral meningitis in immunocompetent hosts, usually occurring in patients with primary genital herpes infection; it may also occur in the absence of clinical genital herpes. Most cases are self-limited. Nonprimary HSV infection is rarely complicated by meningitis. HSV-2 meningitis can recur; it is the most common etiologic agent of Mollaret’s meningitis, a syndrome characterized by recurrent self-limited episodes of meningitis. HSV encephalitis is caused by HSV-1 in the vast majority of cases (Figure 30-2).1,8
607
FIGURE 30-2 Axial and coronal T2-weighted magnetic resonance images showing areas of hyperintensity involving the temporal and inferior frontal lobes (arrows), characteristic of herpes simplex encephalitis. (Courtesy of Southampton General Hospital’s picture library. Reprinted with permission from BMJ Publishing Group Ltd.)
When does Meningitis is an early manifestation of HIV infection, occurring in up to one-fifth of patients upon seroconversion. meningitis usually Other features of primary HIV infection may be present, including lymphadenopathy, rash, dermatitis, occur during the Gastrointestinal disturbances, oral candidiasis, and pharyngitis. Resolution of meningeal symptoms occurs over course of human the course of several weeks. It is important to consider HIV in patients with self-limited “aseptic” meningitis so as to not immunodeficiency miss the diagnosis.7 virus infection? What is the treatment LCMV meningitis should be considered in any patient with meningitis who has a history of contact with rodent for lymphocytic droppings or urine. There is a predilection for the winter months. There is no specific antiviral therapy for LCMV choriomeningitis meningitis; the vast majority of patients experience complete recovery.9 virus meningitis? What are the Viruses that are transmitted via the bite of an arthropod, including mosquitoes, ticks, and flies, are referred to as arboviruses? arboviruses. There are hundreds of such viruses, but only a subset of them cause meningitis, such as West Nile virus, St. Louis encephalitis virus, and California encephalitis viruses (eg, La Crosse virus). Infections generally peak in the summer and early fall. The vast majority of patients are asymptomatic. In symptomatic patients, the most common presentation is a flu-like illness without central nervous system (CNS) involvement after an average incubation period of 3 to 8 days. The most common presentation of CNS involvement is encephalitis, but isolated meningitis can occur. Compared with enteroviruses and herpesviruses, the sensitivity of CSF PCR for arboviruses is significantly lower. Serologies can be complementary. Treatment is supportive.10 What is the prognosis Meningitis is the most common neurologic manifestation of infection with mumps virus, occurring in about 15% of of mumps patients (with male predominance). It can precede or follow parotid swelling; one-half of cases occur in the meningitis? absence of parotitis. The vast majority of patients experience complete recovery.1,9 Other viruses that can cause meningitis include Epstein-Barr virus, cytomegalovirus (in immunocompromised patients), varicella-zoster virus, adenovirus, and human herpes virus 6. 1
608
Typical Bacterial Meningitis
What is the opening pressure of typical bacterial meningitis? What is the characteristic cerebrospinal fluid profile of typical bacterial meningitis? Typical bacterial How is the diagnosis of typical bacterial meningitis definitively made?
What empiric antibiotic regimen is recommended for adult patients with suspected community-acquired bacterial meningitis?
The opening pressure of typical bacterial meningitis is usually elevated (>18 cm H2O).2
Fluid type7 WBC/µL WBC Type Glucose (mg/dL) Protein (mg/dL)
500-20,000 Neutrophils Low 100-700 The diagnosis of typical bacterial meningitis is made when CSF samples demonstrate the presence of bacteria by Gram stain or culture. In most cases, these tests are associated with detection rates of up to 90%; CSF PCR studies can be adjunctive in some cases. Blood cultures are positive in around one-half of cases.11 In adults younger than 50 years of age with community-acquired bacterial meningitis, empiric coverage should include vancomycin (to cover Streptococcus pneumoniae with reduced penicillin sensitivity) and a third-generation cephalosporin (to cover Neisseria meningitidis); in adults older than 50 years of age, the recommendation is vancomycin, a third-generation cephalosporin, and ampicillin (to cover Listeria monocytogenes).5
What are the causes of typical bacterial meningitis?
A common cause of community-acquired pneumonia. Associated with Waterhouse-Friderichsen syndrome (adrenal hemorrhage and hypotension). Vaccination has dramatically reduced the incidence of meningitis caused by this organism. The yield of CSF Gram stain for this organism is only about 30%. This gram-positive organism typically causes meningitis in the context of open head trauma or neurosurgery. These organisms are common causes of nosocomial meningitis.
What is the
Streptococcus pneumoniae.
Neisseria meningitidis.
Haemophilus influenzae.6
Listeria monocytogenes.7
Staphylococcus aureus.
Gram-negative rods.
Streptococcus pneumoniae accounts for more than one-half of bacterial meningitis cases in the industrialized world.
609
treatment for Streptococcus pneumoniae meningitis?
What are the cerebrospinal fluid Gram stain findings in patients with Neisseria meningitidis meningitis?
How common is Haemophilus influenzae meningitis?
What is unique about the cerebrospinal fluid profile in Listeria monocytogenes meningitis compared with the other typical bacterial organisms? What is the treatment for Staphylococcus aureus meningitis? Which gram-negative bacilli are associated with meningitis?
Patients younger than 5 years and older than 60 years are at greatest risk. Empiric antimicrobial therapy includes a cephalosporin (eg, ceftriaxone, cefotaxime, cefepime) plus vancomycin. This regimen, which may be subsequently modified based on sensitivity data, should be continued for a total duration of 2 weeks of intravenous therapy. The administration of adjunctive glucocorticoids before or around the same time as antimicrobial therapy improves mortality and morbidity. Despite optimal therapy, up to one-third of patients with Streptococcus pneumoniae die; up to one-half of survivors experience chronic neurologic sequelae (eg, hearing loss). 2,11 Neisseria meningitidis is a major cause of meningitis worldwide, often occurring in outbreaks. Meningococci are transmitted from person to person via direct contact or droplets. Clinical manifestations of invasive meningococcal disease range from transient bacteremia to a fulminant, life-threatening illness. Meningitis develops in a significant proportion of patients with invasive disease. The majority of these patients develop the characteristic hemorrhagic skin rash (typically petechial, but can be purpuritic or ecchymotic), usually affecting the trunk and lower extremities (Figure 30-3). Gram stain of the CSF demonstrates gram-negative diplococci inside and outside of neutrophils. Empiric antimicrobial therapy includes ceftriaxone or cefotaxime, which can be narrowed to penicillin G in patients with susceptible strains, for a total duration of 7 days in uncomplicated cases. Despite therapy, mortality rate is approximately 10%, and chronic neurologic sequelae develop in about 10% of survivors.2,12,13
FIGURE 30-3 Fulminant petechial and purpuric rash in a patient with meningococcemia. (From Scheld MW, Whitley RJ, Marra CM. Infections of the Central Nervous System. 4th ed. Philadelphia, PA: Lippincott Williams & Wilkens; 2004.)
Invasive disease caused by Haemophilus influenzae type b (Hib) has nearly been eradicated in regions with widespread use of vaccination. However, strains other than Hib are capable of causing meningitis in these regions, and Hib continues to be a major cause of meningitis worldwide. Antimicrobial therapy of choice is a third-generation cephalosporin (eg, ceftriaxone, cefotaxime) for at least 7 days. Despite therapy, mortality rate is about 7%; chronic neurologic sequelae develop in up to one-third of survivors.14 Listeria monocytogenes is a foodborne pathogen that most often causes meningitis in newborns, immunocompromised patients, and the elderly. Compared with other causes of typical meningitis, it is associated with fewer WBCs, lower protein concentration, and a trend toward a lower percentage of neutrophils and less hypoglycorrhachia. Gram-positive rods on CSF Gram stain is highly suggestive, but yield is low (approximately 30%). Antimicrobial therapy of choice is ampicillin for at least 3 weeks; gentamicin is added in patients with severe disease. Despite therapy, mortality rate is as high as 30%, and there is a high rate of chronic neurologic sequelae in survivors.2,15
Staphylococcus aureus causes a small but increasing number of cases of acute bacterial meningitis. It occurs as a result of direct inoculation of the CNS from contiguous infection, trauma, or iatrogenic procedures (eg, neurosurgery), or from hematogenous spread of infection originating outside of the CNS. Antimicrobial therapy of choice is nafcillin for susceptible strains, whereas vancomycin is used for methicillin-resistant Staphylococcus aureus. Despite therapy, up to one-third of patients die.2,16 Escherichia coli, Klebsiella species, and Pseudomonas aeruginosa are among the causes of gram-negative bacillary meningitis. Third-generation cephalosporins are adequate for treating most organisms with the exception of Pseudomonas aeruginosa, which should be treated with ceftazidime, cefepime, or meropenem. The recommended duration of therapy is 3 weeks.2,5
610
Atypical Bacterial Meningitis
How do the cerebrospinal fluid profiles differ in patients with atypical versus typical bacterial meningitis?
Compared with typical bacterial meningitis, atypical bacterial meningitis is associated with the following CSF profile: WBC count tends to be lower with lymphocytic predominance, protein tends to be lower (but remains elevated), and glucose can be normal or low.7
What are the causes of atypical bacterial meningitis?
Common in the developing world, meningitis related to this organism is challenging to diagnose because of its insidious onset with nonspecific symptoms and the insensitivity of available tests. Gram-negative corkscrew-shaped organisms. An antecedent respiratory illness is typical. A 54-year-old man presents with pneumonia and meningismus 3 weeks after helping in the birthing of sheep. This organism is most commonly acquired through the ingestion of contaminated raw milk or unpasteurized cheese. Intracellular organisms carried by arthropods.
Mycobacterium tuberculosis.
Spirochetes.
Mycoplasma pneumoniae.
Coxiella burnetii (Q fever).
Brucella species.
Rickettsia species.
What is the typical Tuberculous meningitis is the least common but most severe form of extrapulmonary tuberculosis (TB). HIV cerebrospinal fluid infection is an important risk factor in adult patients. The presentation is more insidious than typical bacterial profile of tuberculous meningitis, occurring over a period of weeks. Patients typically present with headache and subtle mental status meningitis? changes after a prodrome of low-grade fever, malaise, anorexia, and irritability. The CSF profile resembles fluid type C (see Table 30-1). Early in the course of the illness, there may be a predominance of neutrophils with a gradual shift to lymphocytes over 7 to 10 days. Positive CSF culture confirms the diagnosis, but results take weeks. Ziehl-Neelsen staining for acid-fast bacilli is positive in a minority of cases, although yield may be Increased to approximately 60% with large-volume samples or repeat LPs. CSF PCR has a sensitivity of about 80% but a false- positive rate of about 10%.2,7,17 What are the features Lyme meningitis occurs as a result of invasion of the CNS by Borrelia burgdorferi, typically occurring within a few of meningitis caused months of primary infection. Common symptoms include headache, fatigue, stiff neck, and malaise. Most patients by Borrelia experience antecedent erythema migrans, and many will experience concomitant cranial neuropathy (eg, facial burgdorferi? nerve palsy) or peripheral radiculoneuritis. The CSF is abnormal in virtually all patients with Lyme meningitis, resembling fluid type D (see Table 30-1). CSF culture is positive in <5% of cases; serum and CSF antibody testing is helpful in establishing the diagnosis. The antimicrobial treatment of choice is ceftriaxone for several weeks. Other spirochetes capable of causing meningitis include Treponema pallidum (ie, syphilis) and Leptospira species (ie, leptospirosis).2,18
What is the prognosis of Mycoplasma
Mycoplasma pneumoniae is a common cause of upper and lower respiratory tract infection, particularly in children. CNS involvement occurs in around 7% of patients who are hospitalized for Mycoplasma pneumoniae infection. The
611
system?
of the central nervous
What are the features of Coxiella burnetii meningitis?
What are the features of Brucella meningitis?
What antimicrobial agent is used to treat rickettsial meningitis?
occur. Most patients experience an antecedent respiratory illness. Serologies and PCR from nasopharyngeal or CSF samples can be helpful in establishing the diagnosis. Macrolide antibiotics are the mainstay of treatment. It is not clear whether adjunctive therapyisuch as glucocorticoids improves outcomes in patients with CNS disease. Most patients recover, but many exper ence chronic neurologic sequelae. 19,20 Infection with Coxiella burnetii occurs worldwide and most often develops after exposure to birth fluids of sheep or other mammals, or ingestion of raw milk or fresh goat cheese. The most frequent clinical manifestations include a self-limited flu-like illness, pneumonia, and hepatitis. Meningitis is an uncommon complication but should be considered in patients with exposure history who present with fever, headache, and confusion. CSF is abnormal in most cases, resembling fluid type D (see Table 30-1), except protein may be with in the normal range. Serologic testing is the preferred diagnostic method. Doxycycline is the treatment of choice and most patients fully recover.21,22 Brucellosis is the most common zoonosis in the world; it is endemic to certain regions, including the Mediterranean basin, Persian Gulf, India, and parts of Latin America. Neurobrucellosis occurs in about 5% of cases; meningitis is the most frequent manifestation and may have an acute or chronic presentation. Fever and headache are the most frequent presenting complaints, occurring in most patients. CSF is abnormal in virtually all cases, with lymphocytic pleocytosis (mean 250 cells/µL), elevated protein (range 50-500 mg/dL), and normal or low glucose levels. Optimal antimicrobial treatment requires multiple agents, such as the combination of doxycycline, rifampin, and either ceftriaxone or trimethoprim/sulfamethoxazole.23-25 Rickettsial infections occur in endemic regions worldwide. Tick-borne rickettsial infections that are endemic to the Unites States include Rocky Mountain spotted fever (RMSF), other spotted fever group rickettsioses, ehrlichiosis, and anaplasmosis. Most cases occur between April and September. Meningitis can be a manifestation of late-stage RMSF and, when accompanied by the classic petechial rash, can be confused for meningococcemia. Up to one-fifth of patients with ehrlichiosis develop CNS involvement, including meningitis or meningoencephalitis. CNS involvement in anaplasmosis is rare. Doxycycline is the antimicrobial agent of choice for rickettsial infections.26,27
612
Fungal Meningitis
What is the typical opening pressure of fungal meningitis? What is the characteristic cerebrospinal fluid profile of fungal meningitis? Fungal
The opening pressure of fungal meningitis is typically elevated (>18 cm H2O).2 Fluid WBC/µL WBC Type Glucose Protein Type7 (mg/dL) (mg/dL) 25-500 Lymphocytes or Low or 50-500 neutrophils normal
What are the causes of fungal meningitis?
A 33-year-old man with Cryptococcosis. acquired immunodeficiency syndrome (AIDS) presents with headache and photophobia and is found to have a markedly elevated opening pressure on LP. A 54-year-old woman Histoplasmosis or blastomycosis. presents with symptoms and signs of meningitis months after returning from a family reunion in Memphis, Tennessee. A 28-year-old Filipino man Coccidioidomycosis. presents with fever, headache, and nuchal rigidity 6 weeks after returning home from vacation in Phoenix, Arizona. Meningitis related to this species.28 ubiquitous fungus Aspergillus surprisingly occurs more frequently in immunocompetent hosts than in immunocompromised hosts. Meningitis related to this Candida species. ubiquitous fungus occurs more frequently in immunocompromised patients, those receiving broad-spectrum antibiotics or parental nutrition, and those who have undergone neurosurgical procedures.
Which 2 species of Cryptococcus neoformans is a ubiquitous fungus that typically affects immunocompromised hosts (eg, Cryptococcus cause patients infected with HIV); Cryptococcus gattii is an endemic fungus found in various regions of the world meningitis? (eg, the Northwestern United States) that often affects immunocompetent hosts. Between the 2, Cryptococcus neoformans meningitis is significantly more common.29 How is histoplasmosis of Histoplasmosis is one of the most common fungal infections in the world; it is endemic to the Central and the central nervous system Southeastern United States, Latin America, Africa, and parts of Asia. Most infections are asymptomatic or diagnosed? manifest as self-limited pneumonia. Up to one-fifth of patients with disseminated histoplasmosis develop CNS involvement. Diagnosis is challenging and often requires a variety of tests, including CSF culture (considered the gold standard; at least 10 mL of fluid should be sent with prolonged incubation period); blood cultures; and Histoplasma antigen and antibody testing of the urine, serum, and CSF. Repeat testing is often required, and more invasive testing (eg, brain biopsy) may be necessary. Mortality and relapse are common. 30 How is blastomycosis of the Blastomycosis has been reported around the world; endemic regions of North America include the
613
central nervous system treated?
Mississippi and Ohio River basins and the areas around the Great Lakes and along the St. Lawrence River. The lungs and skin (Figure 30-4) are common sites of infection; CNS involvement accounts for up to 10% of extrapulmonary disease. The most frequent manifestations include headache, focal neurologic deficits, delirium, vision changes, and seizure. CNS blastomycosis should be treated with a lipid formulation of amphotericin B for 4 to 6 weeks followed by an oral azole for at least 1 year. Despite treatment, mortality rates are high.31
FIGURE 30-4 Cutaneous involvement in a patient with disseminated blastomycosis. The initial lesion is an
What are the features of coccidioidal meningitis?
What unique cerebrospinal fluid study can be helpful in the diagnosis of Aspergillus meningitis?
What are the features of Candida meningitis?
inflammatory nodule that enlarges and ulcerates, often resembling pyoderma gangrenosum. Lesions evolve to verrucous or crusted plaques with sharply demarcated serpiginous borders. Dermatology in General Medicine. 8th ed. New York, NY: McGraw-Hill; 2012:882. Copyright McGraw-Hill Education.) (Reprinted with permission from Goldsmith LA, Katz SI, Gilchrest BA, Paller AS, Leffell DJ, Wolff K. Fitzpatrick’s
Coccidioides species are endemic to desert regions of the Western hemisphere, including parts of Arizona, California, New Mexico, and Texas. Pneumonia is the most common clinical manifestation. One-fifth of patients develop disseminated disease, often months or years later. Dissemination occurs more frequently in some groups, such as Filipino patients. Headache is present in the vast majority of patients with coccidioidal meningitis, often described as bilateral, intense, and throbbing; other manifestations include fever, weight loss, nausea, delirium, and focal neurologic deficits. Without treatment, the vast majority of patients die within 2 years; even with treatment, mortality and relapse rates are high. In contrast, most cases of Coccidioides infections involving other organ systems are self-limited.32-34 Aspergillus meningitis is an uncommon clinical entity that occurs more frequently in immunocompetent patients. It usually develops as a result of extension from regional infection (eg, orbit, ear, paranasal sinuses), direct inoculation following a procedure (eg, neurosurgery), or hematogenous spread in intravenous drug users. Compared with other causes of fungal meningitis, the pleocytosis tends to be neutrophilic and protein levels are higher. CSF culture is positive in only one-third of cases. However, the CSF galactomannan antigen test is associated with a sensitivity approaching 90%. Prognosis is generally poor.28 Candida meningitis is uncommon; it occurs more frequently in immunocompromised patients, those receiving broad-spectrum antibiotics or parental nutrition, and those who have undergone neurosurgical procedures. The most common clinical manifestations include fever, headache, delirium, and meningismus. The pleocytosis may be predominantly neutrophilic or lymphocytic. The diagnosis is established with CSF culture; repeat sampling is often required. Treatment requires a prolonged course of combination antifungals (eg, amphotericin B with flucytosine). Despite treatment, mortality rates are high.35
614
Aseptic Meningitis
What is the typical opening pressure of aseptic meningitis? What is the characteristic cerebrospinal fluid profile of aseptic meningitis? Aseptic
The opening pressure of aseptic meningitis is often normal (<18 cm H2O) but may be mildly elevated in some cases. Fluid WBC/µL WBC Glucose Protein Type7 Type (mg/dL) (mg/dL) 5-1000 Lymphocytes or Normal <100 neutrophils
What are the causes of aseptic meningitis?
615
616
617 Case Summary headache and confusion and is found to have fever, nuchal rigidity, et protein. Neuroimaging revealediabnormal signal involving the left A 34-year-old immunocompromised man presents with acute-ons and agitation. An LP showed bloody CSF with normal opening pressure, lymphocytic pleocytos s, normal glucose, and elevated medial temporal lobe.
What is the most likely diagnosis in this patient? Herpes simplex meningoencephalitis.
618
meningitis would fit the meningi is, aseptic meningitis, and most cases of atypical bacterial meningitis. 7 Bonus Questions
Which general types of cerebrospinal fluid profile
bloody cerebrospinal fluid? in this case? What are the causes of
What is the most likely cerebrospinal fluid in this What is the significance of cause of bloody case? the neuroimaging in this case? What evidence suggests the presence of meningoencephalitis
(ratherithan meningitis alone) n this case? How is the diagnosis of meningoencephalitis What is the treatment of herpes simplex virus confirmed? choice for herpes simplex virus meningoencephalitis?
The CSFtprofile in this case (normal opening pressure, mild lymphocytic pleocytosis, mildly elevated protein, and normal glucose) is consistent with viral
CSF appears cloudy with RBC concentrations between 500 and 6000 cells/µL. The differential diagnosis for grossly bloody CSF includes traumatic LP and true CSF erythrocytosis from intracranial hemorrhage, such as subarachnoid hemorrhage and hemorrhagic meningoencephalitis. Trauma from needle insertion is less likelylin this case because the bloody appearance of the fluid didinot decrease with serial sampling as would be expected. Subarachnoid hemorrhage is unlike y given the absence of a history of head trauma, and negat ve imaging. 7 CSF erythrocytosis occurs in approximately 80% of cases of HSV encephalitis and can be an early diagnostic clue. It is thought to be a result of the necrotizing and hemorrhagic nature of the infection. 40
unilateral or asymmetric bilateral high signal involving the medial temporal lobes, insular cortex, and/or orbital surface of the frontal lobes (see Figure 30- MRI can be helpful in the diagnosis of HSV encephalitis, demonstrating characteristic abnormalities in the vast majority of cases. Typical findings include ). In this case, HSV encephalitis is suggested by the presence of abnormal cerebral function early in the course of illness, and temporal lobe abnormalit es o 2 41 neuroimaging. Although HSV meningitis is typically a self-limited illness, HSV encephalitis (either as an isolated entity or together with meningitis)iis a n life-threatening emergency that warrants antiviral therapy. 7
The detection of HSV DNA in CSF by PCR is highly sensitive and specific for the diagnosis of HSV meningoencephalitis. PCR remains positive at least 5 to 7 days after starting therapy. In up to 10% of cases of HSV encephalitis, the CSF may be completely normal. 41
HSV meningoencephalitis is a life-threatening illness that should be treated with intravenous,acyclovir without delay. Treatment reduces mortality rates from >70% to about 20%. Long-term neurologic sequelae (eg, amnesia) can occur in survivors particularly when diagnosis and treatment are delayed. 41
619 Key Points
Meningitis is inflammation of the meninges associated with pleocytosis. Symptoms of meningitis include headache, neck stiffness, neck pain, lethargy, confusion (which is usually no prominent in the
initial course of the illness), nausea, vomiting,tphotophobia, and phonophobia. Physical findings of meningitis include fever, nuchal rigidity,
Kernig’s sign, Brudziński’s sign, papilledema, and other signs of physical findings may be specific to underlying organisms (eg, the Increased intracranial pressure (eg, sixth nerve palsy). Some maculopapular and petechial skin rash of meningococcemia). Encephalitis is inflammation of the brain parenchyma,
characterized by neurologic dysfunction early in the course of Meningoencephalitis is used to describe cases with clinical features Meningitis can be viral, bacterial, fungal, or aseptic. illness. of both meningitis and encephalitis. The CSF profile (eg, opening pressure, WBC count, protein concentration, and glucose concentration) can be useful in distinguishing between the causes of meningitis. CT imaging of the head should be obtained before LP in patient
who are at Increased risk for the presence of a mass lesion with s Increased intracranial pressure.
620
References 1. Logan SA, MacMahon E. Viral meningitis. BMJ. 2008;336(7634):36-40. 2. Longo DL, Fauci AS, Kasper DL, Hauser SL, Jameson JL, Loscalzo J, eds. Harrison’s Principles of Internal Medicine. 18th ed. New York, NY: McGraw-Hill; 2012. Mehndiratta M, Nayak R, Garg H, Kumar M, Pandey S. Appraisal of Kernig’s
- Brudzinski’s sign in meningitis. Ann Indian Acad Neurol. 2012;15(4):287-288. and
-
practice guidelines by the Infectious Diseases Society of America. Clin Infect Dis. Tunkel AR, Glaser CA, Bloch KC, et al. The management of encephalitis: clinical 2008;47(3):303-327.
-
Tunkel AR, Hartman BJ, Kaplan SL, et al. Practice guidelines for the management of bacterial meningitis. Clin Infect Dis. 2004;39(9):1267-1284. Brouwer MC, Tunkel AR, van de Beek D. Epidemiology, diagnosis, and antimi
-
treatment of acute bacterial meningitis. Clin Microbiol Rev. 2010;23(3):467-492.crobial
- WalkertHK, Hall WD, Hurst JW, eds. Clinical Methods: The History, Physical, and Labora ory Examinations. 3rd ed. Boston: Butterworths; 1990.
-
Kupila L, Vuorinen T, VainionpaalR, Hukkanen V, Marttila RJ, Kotilainen P. Etiology of aseptic meningitis and encepha itis in an adult population. Neurology. 2006;66(1):75-80. Meyer HM Jr, Johnson RT, Crawford IP, Dascomb HE, Rogers NG. Central nervous
-
system syndromes of “vital” etiology. A study of 713 cases. Am J Med. 1960;29:334-347. 10. Davis LE, Beckham JD, Tyler KL. North American encephalitic arboviruses. Neurol Clin. 2008;26(3):727-757, ix.
-
Hoffman O, Weber RJ. Pathophysiology and treatment of bacterial meningitis. Ther Adv Neurol Disord. 2009;2(6):1-7. . Heckenberg SG, de Gans J Brouwer MC, et al. Clinical features, outcome, and
12meningococcal genotype in,258 adults with meningococcal meningitis: a prospective cohort study. Medicine (Baltimore). 2008;87(4):185-192. . Manchanda V, Gupta S, Bhalla P. Meningococcal disease: history, epidemiology
13pathogenesis, clinical manifestations, diagnosis, antimicrobial susceptibility and , prevention. Indian J Med Microbiol. 2006;24(1):7-19.
- AgrawaltA, Murphy TF. Haemophiluslinfluenzae infections in the H. influenzae type 15surveillancelof community-acquired Listeria monocytogenes meningitis in adults. BMC b conjuga e vaccine era. J Clin Microbio . 2011;49(11):3728-3732. . Amaya-Vil ar R, Garcia-Cabrera E, Sulleiro-Igual E, et al. Three-year multicenter Infect Dis. 2010;10:324. . Aguilar J, Urday-Cornejo V, Donabedian S, Perri M, Tibbetts R, Zervos M.
16Staphylococcus aureus meningitis: case series and literature review. Medicine (Baltimore). 2010;89(2):117-125.
-
Chin JH. Tuberculous meningitis: diagnostic and therapeutic challenges. Neurol Clin Pract. 2014;4(3):199-205.
-
Pachner AR. Early disseminated Lyme disease: Lyme meningitis. Am J Med. 1995;98(4A):30S-37S; discussion 7S-43S.
-
Bitnun A, Ford-Jones EL,.Petric M, et al. Acute childhood encephalitis and 20infections: summary of cases at the University of Helsinki and review. Clin Infect Dis. Mycoplasma pneumoniae Clin Infect Dis. 2001;32(12):1674-1684. . Koskiniemi M. CNS manifestations associated with Mycoplasma pneumoniae
621
1993;17(suppl 1):S52-S57. 21. Bernit E, Pouget J, Janbon F, et al. Neurological involvement in acute Q fever: a report of 29 cases and review of the literature. Arch Intern Med. 2002;162(6):693-700.
- Kofteridis DP, Mazokopakis EE, Tselentis Y,;Gikas A. Neurological complications of acute Q fever infection. Eur J Epidemiol. 2004 19(11):1051-1054.
-
Pappas G, Akritidis N, Christou L. Treatment of neurobrucellosis: what is known and what remains to be answered. Expert Rev Anti Infect Ther. 2007;5(6):983-990.
-
Pappas G, Papadimitriou P, Akritidis N, Christou L, Tsianos EV. The new global map of human brucellosis. Lancet Infect Dis. 2006;6(2):91-99. . Yetkin MA, Bu ut C, Erdinc FS, Oral B, Tulek N. Evaluation of the c
25presentations inlneurobrucellosis. Int J Infect Dis. 2006;10(6):446-452.linical 26. Biggs HM, Behravesh CB, Bradley KK, et al. Diagnosis and management of tickborne rickettsial diseases: Rocky Mountain spotted fever and other spotted fever Group
rickettsioses, ehrlichioses, and anaplasmosis–United States. MMWR Recomm Rep. 2016;65(2):1-44.
-
Ismail N,;Bloch KC, McBride JW. Human ehrlichiosis and anaplasmosis. Clin Lab 28. Antinori S, CorbellinolM, Meroni L, et al. Aspergillus meningitis: a rare clinical 92 Med. 2010 30(1):261-292. manifestation of centra nervous system aspergillosis. Case report and review of cases. J Infect. 2013;66(3):218-238.
-
Williamson PR, Jarvis JN, Panackal AA, et al. Cryptococcal meningitis: epidemiology, immunology, diagnosis and therapy. Nat Rev Neurol. 2017;13(1):13-24.
-
Hariri OR,tMinasian T, Quadri SA, et al. Histoplasmosis with deep CNS involvement: case presen ation with discussion and literature review. J Neurol Surg Rep. 2015;76(1):e167-e172.
-
Bariola JR, Perry P, Pappas PG, et al. Blastomycosis of the central nervous system: a multicenter review of diagnosis and treatment in the modern era. Clin Infect Dis.
2010;50(6):797-804. 32. Crum NF, Lederman ER, Stafford CM, Parrish JS, Wallace MR.iCoccidioidomycosis: a descriptive survey of a reemerging disease. Clinical characterist cs and current controversies. Medicine (Baltimore). 2004;83(3):149-175.
-
Mathisen G, Shelub A, Truong J, Wigen C. Coccidioidal meningitis:iclinical presentation and management in the fluconazole era. Medicine (Balt more). 2010;89(5):251-284.
-
Vincent T, Galgiani JN, Huppert M, Salkin D. The natural history of coccidioidal meningitis: VA-Armed Forces cooperative studies, 1955-1958. Clin Infect Dis.
1993;16(2):247-254. 35. GoldaniiLZ, Santos RP. Candida tropicalis as an emerging pathogen in Candida 36. Jolles S, Sewell WA, Leighton C. Drug-induced aseptic meningitis: diagnosis and meningit s: case report and review. Braz J Infect Dis. 2010;14(6):631-633. management. Drug Saf. 2000;22(3):215-226.
-
Le Rhun E, Taillibert S, Chamberlain MC. Carcinomatous meningitis:.Leptomeningeal 38. Jarrin I, Sellier P, Lopes A, et al. Etiologies and managementiof aseptic meningitis in metastases in solid tumors. Surg Neurol Int. 2013;4(suppl 4):S265-S288 patients admitted to an internal medicine department. Medic ne (Baltimore). 2016;95(2):e2372.
-
Houllis G, Karachalios M, eds. Meningitis: Causes, Diagnosis and Treatment. New York: Nova Science Publishers; 2012.
622
- WhitleyiRJ, Soong SJ, Linneman C Jr, Liu C, Pazin G, Alford CA. Herpes simplex 41. Sabah M, Mulcahy J, Zeman A. Herpes simplex encephalitis. BMJ. 2012;344:e3166. encephal tis. Clinical Assessment. JAMA. 1982;247(3):317-320.
623
CHAPTER 31
624
Pneumonia
625 Case: A 57-year-old man with shaking chills wA previously healthy 57-year-old man is admitted to the hospitale browntsputum. He has not had contactiwith othertsick individuals. He ith sudden-onset dyspnea and cough over the course of 2 days. H complains of fever and shaking chills. His cough is productive of has no traveled outside of Oregon dur ng the pas 6 years. He does not take any medications and has not had antibiotics recently. Temperature is 38.8°C, heart rate is 102 beats per minute, and respiratory rate is 32 breaths per minute. The patient is diaphoretic and visibly short of breath. There is an area of du lness to percussion
over the left posterior chest with Increased tactilelfremitus. There aree tubular breath sounds, inspiratory rales, and egophony over the sam area. Peripheral white blood cell count is 15.1 K/µL with 82%
polymorphonuclear cells and 15% band forms. Sputum Gram stain inChest radiographs with frontal (A) and lateral (B) views are shown demonstrates the presence of gram-positive diplococci. Figure 31-1.
FIGURE 31-1
What is the most likely cause of pneumonia in this patient?
This chapter reviews the causes of pneumonia in immunocompetent hosts. Immunocompromised hosts are susceptible to a wider spectrum of pathogens, which is outside the scope of this chapter. What is Pneumonia refers to infection of the pulmonary parenchyma with characteristic symptoms, physical findings, and pneumonia? radiographic findings. Acute radiographic findings may be challenging to identify in patients with underlying lung disease.1,2 What are the The clinical presentation of pneumonia may vary depending on host factors (eg, age) and the responsible pathogen. symptoms of Symptoms of pneumonia may include dyspnea, cough, sputum production, pleuritic chest pain, chills, malaise, and pneumonia? confusion.1,2 What are the Physical findings of pneumonia may include fever, tachycardia, tachypnea, cachexia, dullness to percussion, Increased physical tactile fremitus, tubular breath sounds, late inspiratory rales, pleural friction rub, egophony, and whispered pectoriloquy.3 findings of pneumonia? How does the Elderly patients with pneumonia may not have cough, sputum production, or peripheral leukocytosis. Up to one-third of clinical these patients are afebrile on presentation.1 presentation of pneumonia differ in elderly patients?
626
noninfectious Noninfectious conditions that can mimic pneumonia includerpulmonary edema, lung cancer, pulmonary infarction, What Up to one-fifth of patients hospitalized for community-acqui ed pneumonia may have an alternative diagnosis. conditions cryptogenic organizing pneumonia, eosinophilic pneumonia, acute interstitial pneumonia, sarcoidosis, vasculitis, can mimic some of the pulmonary alveolar proteinosis, drug toxicity, and radiation pneumonitis.1 features of pneumonia? How is Most cases of pneumonia occur via microaspiration of upper airway secretions contaminated with microorganisms. Other pneumonia cases occur via inhalation of airborne organisms or particles, hematogenous spread, or as a result of septic pulmonary acquired? emboli (eg, from right-sided endocarditis).2 What tests are Tests to identify specific microbial pathogens in patients with pneumonia include Gram stain and culture of blood, available to sputum, and respiratory secretions, antigen testing (eg, legionella and pneumococcal urinary antigens), and polymerase identify chain reaction (PCR) assays for some pathogens (eg, Mycoplasma pneumoniae, viruses). A sputum sample is considered specific satisfactory when there are >25 polymorphonuclear leukocytes and <10 squamous epithelial cells per lower power field (ie, microbial 100×magnification). Methods of obtaining a respiratory sample include spontaneous expectoration, sputum induction, pathogens in nasotracheal suctioning, and endotracheal aspiration (in patients who require intubation). Despite the availability and use patients with of these tests, a specific pathogen is not identified in a significant proportion of cases, and empiric treatment is necessary.1,4 pneumonia? What cardiac Up to one-quarter of hospitalized patients with community-acquired pneumonia experience myocardial infarction, atrial events are fibrillation, or worsening heart failure. These events are associated with an increase in mortality.1 associated with pneumonia? What are the Pneumonia may be acquired within community or hospital settings. The subcategories of pneumonia acquired in the 2 settings in community setting are community-acquired pneumonia (CAP) and endemic pneumonia. The subcategories of pneumonia which acquired in the hospital setting are hospital-acquired pneumonia (HAP) and ventilator-associated pneumonia (VAP). pneumonia Aspiration pneumonia may occur in either setting. may be acquired?
Why is it important to distinguish between pneumonia acquired in community and hospital settings?
The typical pathogens and resistance patterns differ between community and hospital settings. These differences inform the choice of empiric treatment before a specific pathogen is identified (or when a pathogen is not identified). Early initiation of antimicrobial treatment improves outcomes in patients with pneumonia. If a specific organism is identified, antimicrobial therapy should be narrowed accordingly.1
627
Community-Acquired Pneumonia
What are the 2 general groups of organisms that cause community-acquired pneumonia?
Why is it important to distinguish between typical and atypical community-acquired pneumonia?
CAP can be caused by typical or atypical organisms.
The usual pathogens differ between typical and atypical CAP, which informs the choice of empiric treatment.
628
Community-Acquired Pneumonia Caused by Typical Pathogens
What clinical features are suggestive of typical community-acquired pneumonia? What is the empiric treatment for typical community-acquired pneumonia?
Clinical features that favor typical CAP include acute-onset chills and fever, presentation with septic shock, cough with sputum production, pleuritic chest pain, elevated or depressed peripheral white blood cell count with Increased band forms, dense segmental or lobar consolidation on chest radiography, and serum procalcitonin level ≥0.25 µg/L.1
Outpatients with CAP are generally treated empirically to avoid the cost of diagnostic testing. Most outpatients with CAP without coexisting illness or recent use of antimicrobials may be treated with doxycycline or a macrolide (as long as <25% of pneumococci in the community have high-level macrolide resistance). However, in patients with a clinical syndrome suggestive of a typical organism, empiric treatment with a macrolide may be ineffective and amoxicillin-clavulanate is favored (amoxicillin alone may be ineffective, as there are high rates of β-lactamase production among Haemophilus influenzae and Moraxella catarrhalis isolates). Alternatives include levofloxacin or moxifloxacin. For hospitalized patients with CAP, a β-lactam (eg, ceftriaxone, cefotaxime, or ceftaroline) plus azithromycin may be used empirically; a quinolone (eg, levofloxacin or moxifloxacin) may be used instead. The recommended duration of treatment is 5 to 7 days.1
What organisms cause typical community-acquired pneumonia?
629
Resistant strains of Staphylococcus aureus and Pseudomonas aeruginosa, and other more virulent organisms can cause CAP, but are comparatively less common. The clinical syndrome in patients with pneumonia related to these 630
organisms tends to be more severe. Based on individual risk for drug-resistant pathogens (eg, recent antimicrobial exposure, recent hospitalization, residence in a long-term care facility), empiric coverage of resistant organisms such as Staphylococcus aureus and Pseudomonas aeruginosa should be considered.8,9
631
Community-Acquired Pneumonia Caused by Atypical Pathogens
What clinical features are suggestive of atypical community-acquired pneumonia? What is the empiric treatment for atypical community-acquired pneumonia?
Clinical features that favor atypical CAP include stable cough lasting >5 days, absence of sputum production, normal or mild peripheral leukocytosis, procalcitonin level ≤0.1 µg/L, and the presence of extrapulmonary manifestations.1 Atypical CAP may be treated empirically with a macrolide, doxycycline, or fluoroquinolone.1
What organisms cause atypical community-acquired pneumonia?
Antibiotics are not helpful. A 47-year-old woman presents with mild dyspnea and dry cough and is found to have hemolytic anemia. After returning from a recent cruise, a 76-year-old man presents with dyspnea, cough, and diarrhea and is found to have a temperature of 39.2°C and heart rate of 68 beats per minute. Obligate, intracellular bacteria. Most cases of infection involving this organism are acute, manifesting as pneumonia or hepatitis. However, chronic infection can manifest months or years later, most often as endocarditis. A 34-year-old man who has recently been hunting and skinning rabbits presents with features of atypical pneumonia.
Which viral pathogens cause pneumonia?
How common are extrapulmonary manifestations of Mycoplasma pneumoniae pneumonia? How sensitive is the Legionella urinary antigen test?
Viral pneumonia. Mycoplasma pneumoniae.
Legionella species (Legionnaires’ disease).
Chlamydia species. Coxiella burnetii (Q fever).10
Francisella tularensis (tularemia).
Influenza virus, respiratory syncytial virus, parainfluenza virus, human metapneumovirus, adenovirus, coronavirus, rhinovirus, and Middle East respiratory syndrome-related coronavirus are among the causes of viral CAP. Analyzing respiratory secretions by PCR is the method of choice in identifying viral pathogens. In some cases the virus is the only source of infection, whereas in others there may be secondary bacterial infection (almost one-fifth of patients with bacterial-proven CAP are coinfected with a virus). A neuraminidase inhibitor (eg, oseltamivir) is the treatment of choice for influenza pneumonia and is more effective when initiated early in the course of illness.1 Mycoplasma pneumoniae is a common cause of CAP. Extrapulmonary manifestations are present in around one-quarter of cases and can sometimes be more severe and of greater clinical importance than the pneumonia itself. Extrapulmonary manifestations may involve the blood (eg, autoimmune hemolytic anemia), the heart (eg, pericarditis, myocarditis), the skin (eg, Stevens-Johnson syndrome), and the central nervous system (eg, encephalitis, meningitis, optic neuritis).11 The Legionella urinary antigen test is positive in about 75% of patients with pneumonia caused by Legionella pneumophilia serotype 1 (which is responsible for the vast majority of cases). Sputum PCR and culture with selective media can be used to detect other Legionella species.1,12
632
What are the features of Chlamydia pneumonia?
What are the features of Coxiella burnetii pneumonia?
What is the treatment of choice for pneumonic tularemia?
Chlamydia pneumoniae is an obligate, intracellular bacterium that causes up to one-fifth of cases of CAP. Clinical manifestations range from a mild, self-limited illness to severe forms of pneumonia, particularly in the elderly and in patients with preexisting cardiopulmonary disease. Serologies and PCR may be useful in diagnosis. Chlamydia psittaci should be suspected in patients with recent exposure to birds.13 Q fever is a worldwide zoonotic infection caused by Coxiella burnetii. Infection in humans usually occurs via inhalation of bacteria from air contaminated by the excreta of infected animals, particularly birth fluids. Acute infection is symptomatic in one-half of cases, developing after an incubation period of 2 to 3 weeks. It manifests as a nonspecific illness in association with pneumonia or hepatitis. Cough is frequently present but is nonproductive in one-half of patients. Extrapulmonary symptoms are common, including severe headache, myalgias, and arthralgias. Serologies and serum or whole blood PCR assays may be used to establish the diagnosis. Doxycycline is the treatment of choice.10 Tularemia is a zoonotic infection caused by Francisella tularensis and is found predominantly in the Northern Hemisphere. It is most often acquired from arthropod bites or exposure to animal carcasses, such as rabbits and hares. Pneumonia usually develops days to weeks after the start of nonspecific systemic symptoms. Headache and high fever are common early manifestations, followed by a nonproductive cough with or without pleuritic chest pain. Serologies and PCR assays may be used to establish the diagnosis. Unlike most bacterial causes of atypical pneumonia, most strains of Francisella tularensis are resistant to macrolide agents. An aminoglycoside agent (eg, streptomycin) is the treatment of choice.14
633
Community-Acquired Pneumonia Caused by Endemic Pathogens
What is an endemic pathogen?
Endemic pathogens are confined to particular geographical locations and usually affect both immunocompetent and immunocompromised hosts.
What endemic organisms cause community-acquired pneumonia?
A patient who recently emigrated from Mexico presents with weight loss, night sweats, and hemoptysis and is found to have a right upper lobe cavitary lesion on chest imaging. “Valley fever.” Urine antigen testing can be useful in the diagnosis of pneumonia related to this pathogen, although there is significant cross-reactivity with other fungi. Dermatologic findings, including verrucous and ulcerative lesions (see Figure 30-4), are often present in patients with pneumonia caused by this endemic fungus. Following a trip to British Columbia, Canada, a previously healthy 48-year-old man presents with chronic cough and is found to have a 2-cm, spiculated, solitary pulmonary nodule on chest imaging concerning for lung cancer.
What is the risk of active pulmonary tuberculosis following initial infection?
Mycobacterium tuberculosis (TB).
Coccidioides species (coccidioidomycosis). Histoplasma species (histoplasmosis).
Blastomyces dermatitidis (blastomycosis).15
Cryptococcus gattii (cryptococcosis).
Tuberculosis is common worldwide, particularly in endemic regions. Importantly, TB can also be transmitted from infected hosts in nonendemic areas. A substantial proportion of the world’s population has latent TB and is at risk for active disease. The risk of active disease is approximately 5% in the first 18 months after initial infection, and there is a lifetime risk of around 5% thereafter. Pulmonary TB classically presents with chronic cough, sputum production, hemoptysis, fever, night sweats, loss of appetite, and weight loss. Chest imaging (Figure 31-3) and sputum microscopy and culture are the cornerstones of diagnosing active pulmonary TB. PCR assays and histopathology of biopsy samples may be helpful in some cases.16
634
FIGURE 31-3 Chest radiographs (frontal [A] and lateral [B] views) of a 60-year-old Thai woman with fever,
Where is coccidioidomycosis endemic?
What are the features of acute pulmonary histoplasmosis?
What are the features of pulmonary blastomycosis?
What are the features of pulmonary cryptococcosis?
hemoptysis, and weight loss demonstrating dense posterior right upper lobe consolidation with cavitation, characteristic of pulmonary TB. Sputum contained numerous Mycobacterium tuberculosis organisms. (Courtesy of Cristina Fuss, MD.)
Coccidioidomycosis is endemic to the Southwestern United States (eg, Arizona), Mexico, and some areas of Central and South America (eg, Argentina). There is some evidence that the endemic borders may be widening with global warming and climate change. Symptomatic cases resemble bronchitis or pneumonia, and most are self-limited over a period of weeks. Features that may help distinguish coccidioidomycosis from typical CAP include exposure history, profound fatigue, cutaneous manifestations (eg, erythema nodosum), and subacute time course. Serology is the most widely used diagnostic test but is not sensitive early in the course of disease.17 Histoplasmosis is endemic to the Central and Southeastern regions of the United States, Latin America, Africa, and parts of Asia. Infection is common in endemic areas; most are asymptomatic or manifest as pneumonia. Acute pulmonary histoplasmosis is characterized by fever, malaise, headache, weakness, dry cough, and pleuritic chest pain. Chest radiography typically demonstrates patchy areas of consolidation in one or more lobes; a potential distinguishing feature is the presence of hilar and mediastinal lymphadenopathy. The illness is most often self-limited with prompt resolution of symptoms. However, patients with underlying immunocompromised status or those exposed to a large inoculum of the fungus may develop severe pulmonary infection.18 Blastomycosis has been reported around the world; endemic regions of North America include the Mississippi and Ohio River basins and the areas around the Great Lakes and along the St. Lawrence River. Pneumonia is the most common manifestation of infection. Features include fever, productive cough with or without hemoptysis, myalgias, arthralgias, and pleuritic chest pain. Chest imaging often demonstrates an alveolar or mass-like opacity that can mimic malignancy. Chronic pneumonia develops in a large proportion of untreated patients. Extrapulmonary disease is common and may be an important clue to the diagnosis.15 Pulmonary cryptococcosis ranges from asymptomatic disease to severe pneumonia with respiratory failure, depending on the immune status of the host. Manifestations include cough with or without hemoptysis, fever, malaise, pleuritic chest pain, weight loss, dyspnea, and night sweats. The radiographic abnormality is typically focal and often mistaken for lung cancer. The burden of symptoms is lower in immunocompetent hosts, and tends to resolve over a period of weeks to months even without treatment; however, disseminated disease can occur. Immunocompromised hosts experience rapid-onset pulmonary disease with a heavy burden of symptoms; dissemination is common, particularly to the CNS.19
635
Hospital-Acquired Pneumonia
What is the definition of hospital-acquired pneumonia? How common is hospital-acquired pneumonia? What organisms should be covered in empiric antibiotic regimens for hospital-acquired pneumonia? What is the prognosis of hospital-acquired pneumonia?
HAP is defined as the development of pneumonia in a nonintubated patient after 48 hours of hospitalization.20
HAP occurs at a rate of up to 20 cases per 1000 hospital admissions. Those at highest risk include patients who are elderly, immunocompromised, or have underlying lung disease, and those who have undergone recent surgery, or are receiving enteral feeding through a nasogastric tube.21 Any empiric antibiotic regimen for HAP should at least include coverage of enteric gram-negative bacilli, methicillin-sensitive Staphylococcus aureus, and Pseudomonas aeruginosa. Environmental factors (eg, local antibiogram data) and host factors (eg, drug allergies, severity of illness, risk factors for drug-resistant pathogens) should also guide the choice of empiric therapy. Early empiric treatment is associated with a mortality benefit in patients with HAP and should be initiated as soon as possible after the diagnosis is made. However, there should also be an emphasis on de-escalation based on respiratory and blood culture results when possible, as superfluous treatment increases risk of adverse events (eg, Clostridium difficile colitis).20
In addition to significant increases in hospital length of stay as well as cost of care, HAP is associated with an overall mortality rate of up to 50%. HAP that develops early in the hospital course (<5 days) has a better prognosis than late- onset HAP (≥5 days).21,22
What organisms cause hospital-acquired pneumonia?
Enterobacteriaceae. Enteric gram-negative bacilli. Common causes of Haemophilus influenzae and Streptococcus pneumoniae. community-acquired pneumonia, these organisms tend to cause early-onset hospital-acquired pneumonia. A gram-positive organism Staphylococcus aureus. commonly associated with skin and soft tissue infections and endocarditis. These gram-negative Pseudomonas aeruginosa and Acinetobacter baumannii. organisms often develop resistance to multiple antibiotics.
Which enteric gram-negative bacilli cause hospital- acquired pneumonia?
When is hospital-acquired pneumonia more likely to be caused by the pathogens of community-acquired pneumonia (eg, Haemophilus influenzae or Streptococcus pneumoniae)? When should coverage be provided against methicillin-resistant Staphylococcus aureus (MRSA) in patients with hospital-acquired pneumonia?
Enteric gram-negative bacilli that most frequently cause HAP include Escherichia coli, Klebsiella species, and Enterobacter species. Extended-spectrum β-lactamase activity is becoming increasingly common in these organisms, and there is often fluoroquinolone resistance, necessitating treatment with a carbapenem (eg, meropenem).22 Cases of early HAP (defined as HAP occurring after <5 days of hospitalization) are more likely to be caused by the usual organisms involved in CAP.22
MRSA is prevalent in the hospital setting and requires specific antibiotic therapy (eg, vancomycin). It is reasonable to provide MRSA coverage in patients with HAP if any of the following risk factors are present: (1) prior intravenous antibiotic use within 90 days, (2) hospitalization in a unit where >20% of Staphylococcus aureus isolates are methicillin-resistant, or (3) hospitalization in a unit where prevalence of MRSA is unknown. It is also reasonable to provide MRSA coverage in patients who are known to be carriers of MRSA and those who are severely ill and at high risk for mortality.20
636
What risk factors are associated with multidrug-resistant Pseudomonas aeruginosa? What is the treatment for Acinetobacter baumannii?
Pseudomonas aeruginosa strains are frequently multidrug-resistant. Risk factors include prior use of intravenous antibiotics within 90 days (strongest risk factor) and history of underlying chronic lung disease (eg, COPD, bronchiectasis). 20
Given the high probability of drug resistance, treatment for Acinetobacter baumannii should be driven by susceptibility data. In susceptible strains, a carbapenem or ampicillin-sulbactam is considered first-line therapy. Some strains may only be susceptible to polymyxins (eg, polymyxin B, colistin). In such cases, inhaled colistin may be added to augment therapy.20
637
Ventilator-Associated Pneumonia
What is the VAP is defined as the development of pneumonia 48 or more hours after endotracheal intubation.20 definition of ventilator-associated pneumonia (VAP)? How common is VAP occurs in up to 40% of intubated patients; it is the most frequent ventilator-associated complication and most ventilator-associated frequent ICU-acquired infection. The peak incidence of VAP occurs 5 to 9 days after intubation; cumulative risk is pneumonia? proportional to the duration of intubation.21 What methods are Noninvasive methods to identify an organism in patients with VAP are preferred and include endotracheal available to aid in aspiration and blood cultures (positive in about 15% of cases). Invasive methods include bronchoscopy with the identification of bronchoalveolar lavage (BAL) and protected specimen brush (PSB).20 an organism in patients with ventilator-associated pneumonia? What organisms As with HAP, any empiric antibiotic regimen for VAP should at least include coverage for enteric gram-negative should be covered in bacilli, methicillin-sensitive Staphylococcus aureus, and Pseudomonas aeruginosa. Environmental and host factors empiric antibiotic should also guide the choice of empiric therapy. Early empiric treatment is associated with mortality benefit in regimens for patients with VAP and should be initiated as soon as possible after the diagnosis is made. However, there should ventilator-associated also be an emphasis on de-escalation based on respiratory and blood culture results when possible as superfluous pneumonia? treatment increases risk of adverse events.20 When should It is reasonable to provide MRSA coverage in patients with VAP if any of the following are present: (1) any risk coverage be provided factor for multidrug-resistant pathogens, (2) hospitalization in a unit where >10% to 20% of Staphylococcus aureus against MRSA in isolates are methicillin-resistant, or (3) hospitalization in a unit where prevalence of MRSA is unknown. It is also patients with reasonable to provide MRSA coverage in patients who are severely ill and at high risk for mortality.20 ventilator-associated pneumonia? What are some risk Risk factors for multidrug-resistant pathogens in patients with VAP include the following: (1) prior intravenous factors for antibiotic use within 90 days, (2) presence of septic shock, (3) acute respiratory distress syndrome (ARDS) before multidrug- resistant VAP, (4) ≥5 days of hospitalization before VAP, and (5) acute renal replacement therapy before VAP.20 pathogens in patients with ventilator-associated pneumonia? What is the VAP is associated with an overall mortality rate of up to 60%.21 prognosis of ventilator-associated pneumonia?
What organisms cause ventilator-associated pneumonia?
This group of organisms can develop resistance to most β-lactam antibiotics because of enzymes called extended-spectrum β-lactamases. The main gram-positive pathogen responsible for VAP. Some authorities recommend the initial use of 2 antibiotic agents to cover this organism. This organism is typically of low virulence outside the hospital but can easily become multidrug-resistant and in some cases is only susceptible to polymyxins.
How common is enteric gram-negative bacilli ventilator-associated pneumonia in the United States? How common is Staphylococcus aureus ventilator-associated pneumonia in the United States? How common is Pseudomonas aeruginosa ventilator-associated pneumonia in the United States? How common is Acinetobacter baumannii ventilator-associated pneumonia in the United States?
Enteric gram-negative bacilli.
Staphylococcus aureus. Pseudomonas aeruginosa.20 Acinetobacter baumannii.20
Enteric gram-negative bacilli cause up to 40% of VAP in the United States.20 Staphylococcus aureus causes up to 30% of VAP in the United States.20 Pseudomonas aeruginosa causes up to 20% of VAP in the United States.20 Acinetobacter baumannii causes up to 10% of VAP in the United States.20
638
639
Aspiration Pneumonia
How is the mechanism of aspiration pneumonia different from the other types of pneumonia? What is aspiration pneumonitis?
What are the risk factors for aspiration pneumonia?
Does aspiration pneumonia occur in the community or hospital setting? What are the clinical characteristics of aspiration pneumonia?
Which parts of the lungs are typically involved in aspiration pneumonia? What methods are available to aid in identifying an organism in patients with aspiration pneumonia? What is the empiric treatment for aspiration pneumonia?
Most types of pneumonia occur via microaspiration. Organisms such as Streptococcus pneumoniae and Haemophilus influenzae are relatively virulent, so even a small inoculum can result in pneumonia. In comparison, aspiration pneumonia typically results from the macroaspiration of less virulent pathogens. A large inoculum of organisms is generally required to cause infection.
Aspiration pneumonitis is characterized by respiratory distress related to the aspiration of toxic material (eg, gastric acid) into the lower airways with an associated inflammatory response, independent of infection. Most patients with aspiration pneumonitis experience rapid recovery with clearing of radiographic abnormalities. However, some patients develop secondary infection and/or acute respiratory distress syndrome.23 Healthy patients frequently aspirate during sleep without consequence. For aspiration pneumonia to develop, usually the normal defense mechanisms that protect the lower airways (eg, cough, ciliary action, alveolar macrophages) are impaired, or a large inoculum of organisms is aspirated. Predisposing conditions include chronic neurologic disorders (eg, stroke), esophageal disorders, impaired consciousness, alcoholism, illicit drug abuse, use of acid-suppressive medications, vomiting, instrumentation of the respiratory tract (eg, bronchoscopy), and poor dentition.4,24 Aspiration pneumonia occurs in both community and hospital settings. The pathogens in these environments differ because hospitalized patients acquire oropharyngeal colonization with nosocomial organisms. Aspiration pneumonia that develops in the community tends to be caused by organisms that are part of the normal oral flora, such as anaerobes and Streptococcus species. Aspiration pneumonia that develops in the hospital can additionally include other organisms such as gram-negative bacilli and Staphylococcus aureus.4,25
The presentation of aspiration pneumonia depends on the organisms involved. Some pathogens (eg, gram-negative bacilli, Staphylococcus aureus) cause an acute presentation that is difficult to clinically distinguish from CAP or HAP but should be suspected in patients with predisposing risk factors. When anaerobes are involved, patients experience more indolent symptoms with weeks to months of malaise, low-grade fever, cough, and weight loss. Over time, complications such as necrotizing pneumonia (multiple cavities ≤1 cm in diameter), lung abscess (1 or more cavities >1 cm in diameter that communicate with a bronchus) (Figure 31-4), or empyema may develop. These complications may lead to putrid sputum and hemoptysis. Nonanaerobes capable of these destructive complications include Staphylococcus aureus, Klebsiella pneumoniae, and Pseudomonas aeruginosa.4
FIGURE 31-4 Chest radiograph showing a large abscess of the left lung with an air-fluid level (arrow). (From Mulholland MW, et al. Greenfield’s Surgery, Scientific Principles and Practice. 4th ed. Philadelphia, PA: Lippincott Williams & Wilkins; 2005.)
The right lung is most often involved in aspiration pneumonia because the right mainstem bronchus is wider and straighter than the left. In general, the dependent portions of the lungs are most often involved. In patients who aspirate in the recumbent position, these areas include the posterior segments of the upper lobes and the apical segments of the lower lobes. In patients who aspirate in the upright position, these areas include the basal segments of the lower lobes.26
Gram stain and culture are the methods of choice for identifying specific organisms involved in aspiration pneumonia. Expectorated sputum is not suitable for anaerobic culture because anaerobes are part of normal oral flora and contaminate samples. Transtracheal aspiration and bronchoscopy with BAL are techniques that bypass the normal flora of the upper respiratory tract and may provide more reliable specimens for anaerobic media. When present, an empyema provides an excellent source for fluid aspiration and identification of a specific organism. Blood cultures are rarely positive in patients with anaerobic pneumonia but may be helpful in cases involving other organisms.4
Empiric treatment for aspiration pneumonia should take into account the setting in which it occurred. Community-acquired aspiration pneumonia should be treated with agents that cover anaerobes and other constituents of normal oral flora, such as amoxicillin-clavulanate (or ampicillin-sulbactam for patients requiring parenteral therapy). Hospital-acquired aspiration pneumonia may benefit from additional coverage of resistant organisms (eg, Pseudomonas aeruginosa) with an agent such as piperacillin-tazobactam; MRSA coverage may be added for patients with risk factors.4
What organisms cause aspiration pneumonia?
640
These organisms are part of normal oral flora and cause aspiration pneumonia in the community and hospital settings. These organisms are often responsible for aspiration pneumonia in hospitalized patients.
What are the features of aspiration pneumonia caused by anaerobes?
Which streptococci cause aspiration pneumonia? Which gram-negative bacilli cause aspiration pneumonia? What are the features of Staphylococcus aureus pneumonia?
Anaerobes and Streptococcus species.
Gram-negative bacilli and Staphylococcus aureus.
Anaerobic organisms are part of normal oral flora and often cause aspiration pneumonia in the community and hospital settings. These pathogens include species of Peptostreptococcus, Bacteroides, Prevotella, and Fusobacterium. Aspiration pneumonia caused by anaerobes tends to be insidious in onset and leads to lung abscess and empyema formation, although Fusobacterium necrophorum is particularly virulent. Antimicrobial therapy should be prolonged for patients with necrotizing pneumonia, lung abscess, or empyema. In the case of empyema, evacuation of the pleural space is also necessary.4 Viridans streptococci are major constituents of normal oral flora and are important causes of aspiration pneumonia in both community and hospital settings. Other streptococci are capable of causing aspiration pneumonia, including Streptococcus pyogenes.4 Hospitalized patients often acquire oropharyngeal colonization with gram-negative bacilli, such as Klebsiella pneumoniae, Enterobacter species, Serratia species, Pseudomonas aeruginosa, Escherichia coli, and Proteus species. Unlike anaerobes, these organisms tend to produce fulminant pneumonia.4 Staphylococcus aureus pneumonia is common in the hospital setting, particularly after aspiration events. Risk factors include advanced age, prolonged hospitalization, underlying lung disease, prior antibiotic therapy, and surgery. The presentation is typically acute, with a tendency toward necrotizing pneumonia, bacteremia, and septic shock. Gram stain and culture of expectorated sputum are sensitive for the presence of Staphylococcus aureus. Blood cultures are also helpful in making the diagnosis. Despite treatment, the mortality rate is high.4,27
641 Case Summary A previously healthy 57-year-old man presents with acute-onset dyspnea, productive cough, and rigors and is found to have feve
focal findings on lung examination, peripheral leukocytosis, andr, abnormal findings on chest imaging.
What is the most likely cause of pneumonia in this patient? Streptococcus pneumoniae.
642
Bonus Questions
pneumoniaeithe mosts the presence of lobar consolidation on chest imaging (see Figure 31-1). Typical CAP is most often caused by Streptococcus pneumoniae, Haemophilus influenzae, Why is Streptococcus This case is most suggestive of typical CAP, given the acute-onset of symptoms, productive cough, peripheral leukocytosis with Increased band forms, and case? likely organ sm in thi and Moraxella catarrhalis. The Gram stain in this case demonstrates gram-positive diplococci, which is consistent with Streptococcus pneumoniae. What is egophony? Egophony means “goat sound” (from the Greek “ego”), and refers to the change in timbre of sound as it travels through an area of consolidation. Whenent auscultating the chest, ask the patient to say the word “bee.” Egophony is present if the “e” sounds more like “a.” The bleating of a Nubian goat is an excell . What is whisp Pectoriloquy means “chest speaking.” When auscultating the chest, ask the patient to whisper a phrase (such as “66 whiskeys, please”). Normal, aerated lung example of this sound 3 pectoriloquy? ered acts as a filter, making these whispered words garbled and undecipherable. If the words are clearly discernable, then pectoriloquy is present and indicative of underlying nonaerated lung (eg, consolidation, tumor, compressed lung). 3 What findings are present The chest radiograph in this case (see Figure 31-1) demonstrates a large consolidation with air bronchograms filling much of the left upper lobe. There is no on the chest radiograph associated pleural effusion. in this case? Does the patient in this r Factors that increase the likelihood of resistant organisms in CAP include recent antimicrobial exposure, residence in a long-term care facility, tube feeding, case have risk factors fo prior infection with a drug-resistant organism (within 1 year), recent hospitalization, chronic lung disease, poor functional status, immunocompromised drug-resistant status, and chronic hemodialysis. Based on the information provided, the patient in this case does not appear to be at Increased risk for drug-resistant organisms? organisms.8,9 How should the patient Hospitalized patients with CAP without risk factors for drug-resistant pathogens can be treated with an antipneumococcal fluoroquinolone (eg, levofloxacin) in this case be treated? or a combination of a β-lactam agent (eg, ceftriaxone) plus a macrolide (eg, azithromycin). Given the signs of systemic toxicity in this case (such as fever, tachycardia,itachypnea, and leukocytosis), it would be reasonable to treat with intravenous antibiotics until there is clinical improvement before transitioning What is the prognosis of The overall 30-dayrmortality rate in hospitalized patientsrwith CAP is approximately 10%. The 30-day readmission rate is about 20%. Elderly patients may to oral antib otics. 28 pneumonia requiring community-acquired require months to ecover, and some never return to thei previous state of health. 1 hospitalization?
643 Key Points
characteristic symptoms, physical findings, and radiographicwith Pneumonia refers to infection of the pulmonary parenchyma findings. Symptoms of pneumonia include dyspnea, cough, sputum
production, pleuritic chest pain, chills, malaise, and confusion. fremitus, tubular breathlsounds, inspiratory rales, pleural friction Despiteta cadre of available diagnostic tests, a specific pathogen Pneumonia can be acquired in the community or in the hospital. setting are community-acquired pneumonia and endemic Community-acquired pneumonia can be caused by typical or systemic toxicity, whereas atypical pathogens aretassociated with The subcategories of pneumonia acquired in the hospital setting pneumonia. Physical findings of pneumonia include fever, tachycardia, tachypnea, cachexia, du lness to percussion, Increased tactile rub, egophony, and whispered pectoriloquy. may no be identified in many cases of pneumonia. Empiric treatment is often necessary. The subcategories of pneumonia acquired in the community pneumonia. atypical pathogens. Typical bacterial pneumonia tends to present acu ely with indolent symptoms and signs. are hospital-acquired pneumonia and ventilator-associated Hospital-acquired pneumonia is defined as pneumonia occurring after 48 or more hours of hospitalization in nonintubated patients. occurring after 48 or more hours of endotracheal intubation.tal The common pathogens in each category of pneumonia are Ventilator-associated pneumonia is defined as pneumonia Aspiration pneumonia occurs in both community and hospi settings. distinct and affect the choice of empiric antimicrobials. Environmental factors (eg, ocal antibiogram data) and host facto (eg, drug allergies, severitylof illness, presence of risk factors for rs therapy. drug-resistant pathogens) should also be used to guide empiric
644
645
References 1. Musher DM, Thorner AR. Community-acquired pneumonia. N Engl J Med. 2014;371(17):1619-1628.
-
Prina E, Ranzani OT, Torres A. Community-acquired pneumonia. Lancet. 2015;386(9998):1097-1108.
-
Sapira JD. The Art & Science of Bedside Diagnosis. Baltimore, MD: Urban & Schwarzenberg; 1990.
-
Finegold SM. Aspiration pneumonia. Rev Infect Dis. 1991;13(suppl 9):S737-S742.
-
King P. Haemophilus influenzae and the lung (Haemophilus and the lung). Clin Transl Med. 2012;1(1):10.
-
Longo DL, Fauci AS, Kasper DL, Hauser SL, Jameson JL, Loscalzo J, eds. Harrison’s Principles of Internal Medicine. 18th ed. New York, NY: McGraw-Hill; 2012.
-
Verduin CM, Hol C, Fleer A, van Dijk H, van Belkum A. Moraxella catarrhalis: from emerging to established pathogen. Clin Microbiol Rev. 2002;15(1):125-144.
-
Shorr AF, Zilberberg MD, Reichley R, et al. Validation of a clinical score for assessing the risk of resistant pathogens in patients with pneumonia presenting to the
emergency department. Clin Infect Dis. 2012;54(2):193-198. 9. drug resistance in pneumonia clinical prediction score. Antimicrob Agents Chemother. Webb BJ, Dascomb K, Stenehjem E, et al. Derivation and multicenter validation of the 2016;60(5):2652-2663.
- Anderson A, Bijlmer H, Fournier PE, et al. Diagnosis and management of Q fever– United States, 2013: recommendations from CDC and the Q Fever Working Group.
MMWR Recomm Rep. 2013;62(RR-03):1-30. 11. Waites KB, Talkington DF. Mycoplasma pneumoniae and itstrole as a human 12. Yu VL, Plouffe JF, Pastoris MC, et al. Distribution of Legionella species and pathogen. Clin Microbiol Rev. 2004;17(4):697-728, table of con ents. serogroups isolated by culture in patients with sporadic community-acquired
legionellosis: an international collaborative survey. J Infect Dis. 2002;186(1):127-128. 13. Blasi F, Tarsia P, Aliberti S. Chlamydophila pneumoniae. Clin Microbiol Infect. 2009;15(1):29-35.
-
Thomas LD, Schaffner W. Tularemia pneumonia. Infect Dis Clin North Am. 2010;24(1):43-55.
-
Saccente M, Woods GL. Clinical and laboratory update on blastomycosis. Clin Microbiol Rev. 2010;23(2):367-381.
-
Zumla A, Raviglione M, Hafner R, von Reyn CF. Tuberculosis. N Engl J Med. 2013;368(8):745-755.
-
Malo J, Luraschi-Monjagatta C, Wolk DM, Thompson R, Hage CA, Knox KS. Update on the diagnosis of pulmonary coccidioidomycosis. Ann Am Thorac Soc.
2014;11(2):243-253. 18. Kauffman CA. Histoplasmosis: a clinical and laboratory update. Clin Microbiol Rev. 2007;20(1):115-132.
- Shirley RM, Baddley JW. Cryptococcal lung disease. Curr Opin Pulm Med. 2009;15(3):254-260. . Kalil AC, Metersky ML, Klompas M, et al. Management of adults with hospital-
20acquired and ventilator-associated pneumonia: 2016 Clinical Practice Guidelines by the Infectious Diseases Society of America and the American Thoracic Society. Clin Infect
646
Dis. 2016;63(5):e61-e111. 21ventilator-associated pneumonia: recent advances in epidemiology and management. . Barbier F, Andremont A, Wolff M, Bouadma L. Hospital-acquired pneumonia and Curr Opin Pulm Med. 2013;19(3):216-228.
-
Kieninger AN, Lipsett PA. Hospital-acquired pneumonia: pathophysiology, diagnosis, and treatment. Surg Clin North Am. 2009;89(2):439-461, ix.
-
Mendelson CL. The aspiration of stomach contents into the lungs during obstetric anesthesia. Am J Obstet Gynecol. 1946;52:191-205. . Taylor JK, Fleming GB, Singanayagam A, Hill AT, Chalmers JD. Risk factors for
24aspiration in community-acquired pneumonia: analysis of a hospitalized UK cohort. Am J Med. 2013;126(11):995-1001.
-
Takayanagi N, Kagiyama N, Ishiguro T, Tokunaga D, Sugita Y. Etiology and outcome of community-acquired lung abscess. Respiration. 2010;80(2):98-105.
-
Marik PE. Aspiration pneumonitis and aspiration pneumonia. N Engl J Med. 2001;344(9):665-671. . Gonzalez C, Rubio M Romero-Vivas J, Gonza ez M, Picazo JJ Bacteremic pneumonia
27due to Staphylococcus,aureus: a comparison ofldisease caused.by methicillin-resistant and methicillin-susceptible organisms. Clin Infect Dis. 1999;29(5):1171-1177.
- Mandell LA, Wunderink RG, Anzueto A, et al. Infectious Diseases Society of community-acquired pneumonia in adults. Clin Infect Dis. 2007;44(suppl 2):S27-S72. America/American Thoracic Society consensus guidelines on the management of
647